Sunteți pe pagina 1din 64

DERMATOLOGY

Section A
1.
Anatomy of Skin, Cosmetics and Skin Care.........................................................123
2.
Appendageal Disorders........................................................................................125
3.
Pigmentary Disorders..........................................................................................129
4.
Allergic Disorders and Dermatitis (Eczema).........................................................130
5.
Papulosquamous Disorders.................................................................................137
6.
Papulovesicular/Vesicobullous Disorder and Immunologically-
Mediated Diseases...............................................................................................142
7. Systemic Diseases with Prominent Cutaneous Features,
Photosensitivity Disorders and Cutaneous Drug Eruptions..................................148
8. Leprosy (Hensen’s Disease).................................................................................154
9. Fungal Infections, Scabies and Pediculosis..........................................................160
10. Venereal (Sexually Transmitted) Diseases...........................................................165
11. Genodermatoses .................................................................................................170
12. Malignant and Deficiency Diseases of Skin..........................................................173

Section B
Practice Questions.........................................................................................176
(Comprising of Questions from Recent Exams and NEET Pattern Questions)
Section A

1. ANATOMY OF SKIN,
COSMETICS AND SKIN CARE

A. General Anatomy of Skin


B. Cosmetics and Skin Care
124 Jaypee’s Triple A

ANATOMY OF SKIN, COSMETICS AND SKIN CARE (QUESTIONS)

A. GENERAL ANATOMY OF SKIN d. Lines of development


Ref: Rook’s 8/e p15.6 - 15.7, 18.2/4; Millington & Wilkinson 1/e p18-19;
1. Which among the following is the most superficial layer of Neurocuteneous disorders-Ruggieri (Springer 2008)-364
skin: (Kerala PG 2010)
a. Stratum corneum
b. Stratum basale B. COSMETICS AND SKIN CARE
c. Stratum granulosum
d. Stratum spinosum 6. Chemical peeling is done by all except: (AIIMS Nov, 2010)
a. Trichloroacetic acid
2. A skin lesion with small fluid-filled blisters less than 0.5 cm
b. Phosphoric acid
in diameter belongs to: (AP 2011)
c. Carbolic acid
a. Pustule
d. Kojic acid
b. Abscess
Ref: Roxburgh 18/e p347; Neena Khanna 4/e p418
c. Vesicle
d. Bulla Ref: Neena Khanna 4/e p7 7. A mother brought her child who has got a vascular plaque
like lesion over the lateral aspect of forehead mainly
3. True about corneum lucidum: (PGI June 09)
involving ophthalmic and maxillary division of trigeminal
a. Sandwitched b/w s. spinosum & s. granulosum
nerve. Mother says that the lesion remains unchanged
b. Sandwitched b/w s. corneum & s. granulosum
since birth. Also mother gives a history that the child is on
c. Contain hair follicle
valproate for seizure disorder. The probable diagnosis is:
d. Also k/a prickle cell layer
(AIIMS May 12)
e. Contain degenerated cells Ref: Roxburgh 18/e p2-6
a. Tuberous sclerosis
4. Which layer of epidermis is underdeveloped in the VLBW b. Infantile hemangioma
infants in the initial 7 days: (AIIMS Nov 02) c. Sturge weber syndrome
a. Stratum germinativum d. Incontinentia pigment
b. Stratum granulosum Ref: Roxburg 18/e p206; Neena 4/e p372
c. Stratum lucidum
8. Child with erythematous non blanching bosselated lesion
d. Stratum corneum Ref: Roxburgh 18/e p2-6
on right side of face, Rx is: (AIIMS May 2011)
5. Lines of Blaschko represent: (AI 2011) a. Erbium laser
a. Lines along lymphatics b. Nd-YAG laser
b. Lines along blood vessels c. Flash light pumped dye laser
c. Lines along nerves d. Q ruby laser
Ref: Rook’s 8/e p78.5 -78.7, 18.63 – 18.64;
Fitzpatrick’s 7/e p2374-76; 1652-56
DERMATOLOGY

Ans. 1. a. Stratum corneum 2. c. Vesicle 3. b. Sandwitched b/w s.… 4. d. Stratum corneum


5. d. Lines of development 6. b. Phosphoric acid 7. c. Sturge weber 8. c. Flash light pumped...
2. APPENDAGEAL DISORDERS

A. Acne Vulgaris
B. Disorders of Sweat Glands
C. Disorders of Hair
D. Disorder of Nails
E. Miscellaneous
126 Jaypee’s Triple A

APPENDAGEAL DISORDERS (QUESTIONS)

treatment would be: (AI 2006)


A. ACNE VULGARIS a. Acitretin
b. Isotretinoin
1. Not used in acne: (AP 2012)
c. Doxycycline
a. Benzoyl peroxide
d. Azithromycin Ref: Neena Khanna 4/e p118
b. Isotretinoin
c. Adapalene 9. A teenager girl with moderate acne is also complaining of
d. Anthralin Ref: Roxburgh 18/e p169 irregular menses. Drug of choice will be:
a. Oral isotretinon (AIIMS Nov 2010)
2. Causative factor for acne are all except:
b. Oral acitretin
a. Androgen (PGI 2000)
c. Oral minocycline
b. Only food
d. Cyproterone acetate Ref: Neena Khanna 4/e p119
c. Bacterial contamination
d. Hypercornification of duct 10. A 17 year old girl with Acne has been taking a drug for
e. Lipophilic yeast the last two years. She now presents with blue black
Ref: Neena Khanna 4/e p110-116 pigmentation of nails. The likely medication causing the
above pigmentation is: (AI 2010)
3. Causative factor for acne are following except:
a. Tetracycline
a. Androgen (SGPGI 2000)
b. Minocycline
b. Only food
c. Doxycycline
c. Keratin
d. Azithromycin Ref: Neena Khanna 4/e p118
d. Cell nucleus
Ref: Neena Khanna 4/e p110-113 11. Acne vulgaris is due to involvement of:
a. Sebaceous glands (PGI June 06, AIIMS 01)
4. Treatment of acne: (PGI June 2005)
b. Eccrine glands
a. 13 cis retionol
c. Pilosebaceous glands
b. Minocycline/tetracycline
d. Apocrine glands
c. Erythranycin
e. Sweat glands Ref: Neena Khanna 4/e p109
d. Dapsone
e. Rifampicin 12. Rhinophyma is (a complication of): (AI 97, UP 04, AP 98)
Ref: Neena Khanna 4/e p117-118 a. Glandular form of acne rosacea (PGI 96)
b. Form of acne vulgaris
5. Treatment of acne vulgaris may include all except:
c. Affects the scalp
a. Cryotherapy (PGI 96, AIIMS 03)
d. A form of dermatofibroma Ref: Roxburgh 18/e p174
b. Estrogens
c. UV light 13. A 40-year-old woman presents with a 2 year old h/o
d. Androgens Ref: Neena Khanna 4/e p117-119 erythematous papulo pustular lesions on convexities of the
face. There in a background of erytherma & telengiectasia.
6. A patient presented with multiple nodulocystic lesions on
The most likely diagnosis is:
the face. The drug of choice is : (AIIMS May 02)
a. Acne vulgaris (AI 2005)
a. Retinoids
b. Rosacea
b. Antibiotics
c. SLE
DERMATOLOGY

c. Steroids
d. Polymorphic light eruption
d. UV light Ref: Neena Khanna 4/e p117-119
Ref: Harrison 18/e p404; Roxburgh 18/e p172
7. Treatment of choice for acne vulgaris: (PGI Dec 05)
a. Minocycline for inflammatory acne
b. Retinoids for comedonal acne B. DISORDERS OF SWEAT GLANDS
c. Etretinate
d. Rifampicin 14. True about apocrine gland is all except: (PGI June 09)
e. Dapsone a. Modified sweat gland
Ref: Neena Khanna 4/e p117-119 b. Modified sebaceous gland
8. A 24-year-old unmarried women has multiple nodular, c. Present in groin & axilla
cystic, pustular and comadonic lesions on face, upper d. Infection is k/a hydradenitis suppurativa
back and shoulders for 2 years. The drug of choice for her Ref: Neena Khanna 4/e p127

Ans. 1. d. Anthralin 2. b. Only food 3. b. Only food 4. a, b, and c


5. d. Androgens 6. a. Retinoids 7. a and b 8. b. Isotretinoin
9. d. Cyproterone acetate 10. b. Minocycline 11. c. Pilosebaceous glands 12. a. Glandular form…
13. b. Rosacea 14. b. Modified sebaceous
Appendageal Disorders 127
15. Miliaria is a disorder of: (PGI Dec 07) 24. A female patient presents with diffuse alopecia to you.
a. Sebaceous glands She had suffered from typhoid fever 4 months back. Most
b. Apocrine glands probable diagnosis is: (AIIMS Nov 07)
c. Merocrine glands a. Androgenetic alopecia
d. Holocrine glands b. Telogen effluvium
e. Eccrine glands Ref: Neena Khanna 4/e p126 c. Anagen effluvium
16. Fox Fordyce Disease effects: d. Alopecia areata Ref: Roxburgh 18/e p285
a. Sebaceous glands 25. A 30-year-old female developed diffuse hair loss 3 months
b. Eccrine glands after delivery of her first child. The probable diagnosis is:
c. Apocrine glands a. Androgenic alopecia (PGI 01, SGPGI 03)
d. Any gland Ref: Neena Khanna 4/e p123 b. Endocrinal alopecia
c. Telogen effluvium
d. SLE Ref: Roxburgh 18/e p285
C. DISORDERS OF HAIR 26. Cicatrising alopecia with perifollicular blue-gray patches
hyperpigmentation is most commonly associated with:
17. This phase in hair cycle determines the length to which hair
a. Pitting of nails (AI 2011)
in different body areas can grow: (Karnataka 2011)
b. Whitish lesions in the buccal mucosa
a. Anagen
c. Arthritis
b. Catagen
d. Discoid plaques on the face Ref: Harrison 18/e p407
c. Telogen
d. Kenogen Ref: Roxburgh 18/e p9 27. Cicatrisial alopecia is seen in: (PGI Dec 04, 07)
a. DLE
18. This is a cause for scarring alopecia: (Karnataka 2011)
b. Psoriasis
a. Alopecia areata
c. Alopecia areata
b. Aplasia cutis
d. Lichen planus
c. Trichotillomania
e. SLE Ref: Neena Khanna 4/e p136
d. Sarcoidosis Ref: Harrison 18/e p59-407-399
28. Scarring alopecia is seen in: (AI 2009)
19. The characteristic clinical sign of alopecia areata is:
a. T. capitis
a. Exclamatory mark hair (Kerala PG 2010)
b. Androgenic alopecia
b. Decreased hair diameter
c. Alopecia areata
c. Absence of hair follicle
d. Lichen planus Ref: Roxburgh 18/e p287
d. A scaly patch of alopecia Ref: Neena Khanna 4/e p130
29. All of the following are causes of cicatrizing alopecia
20. Which of the following conditions causes alopecia without
except: (AIIMs Nov 07)
scarring?: (DP PGMEE 2010)
a. Lichen planus
a. LE
b. Discoid lupus erythematosus
b. Herpes zoster
c. Alopecia areata
c. Alopecia areata
d. Lupus vulgaris Ref: Neena Khanna 4/e p136
d. Lichen planus pillaris Ref: Neena Khanna 4/e p130, 131
30. Alopecia aerata is: (SGPGI 2004)
21. In Fox Fordyce disease, true is/are: (PGI 2002)
a. Cicatricial scar
a. Common in adult woman
b. Non cicatricial scar
b. Bullous lesions are common
c. Fungal infection
c. Common in areola & axilla

DERMATOLOGY
d. None Ref: Neena Khanna 4/e p30
d. Associated with other malignancies Ref: Internet
31. Non-circatrical alopecia is seen in: (PGI May 11)
22. Anagen phase of the hair indicates: (AIIMS May 06)
a. Alopecia areata
a. The phase of activity and growth
b. Androgenetic alopecia
b. The phase of transition
c. Pseudopalade
c. The phase of resting
d. DLE
d. The phase of degeneration Ref: Roxburgh 18/e p9
e. SLE Ref: Neena Khanna 4/e p130, 133, 136
23. The time period that elapses between the physic emotional
32. Exclamation mark hairs is seen in: (Bihar 06, Kerala 01)
stress and the hair loss is about:
a. Alopecia areata (AIIMS Nov 05)
a. 21 days (JIPMER 99, DNB 01)
b. Traumatic alopecia
b. 30 days
c. Lichen planus
c. 3 months
d. All
d. 6 months
Ref: Neena Khanna 4/e p130

Ans. 15. e. Eccrine glands 16. c. Apocrine glands 17. a. Anagen 18. b. Aplasia cutis
19. a. Exclamatory mark... 20. c. Alopecia areata 21. a and c 22. a. The phase of activity
23. c. 3 months 24. b. Telogen effluvium 25. c. Telogen effluvium 26. b. Whitish lesions....
27. a, d and e 28. d. Lichen planus 29. c. Alopecia areata 30. b. Non cicatricial scar
31. a and b 32. a. Alopecia areata ....
128 Jaypee’s Triple A
33. Male with patchy loss of scalphair and grey hair in the c. Tinea unguium
eyebrows and beard, diagnosis is: d. Alopecia areata Ref: Neena Khanna 4/e p143
a. Anagen effluvium (AI 08) 42. Wrong statement is: (AI 2000)
b. Alopecia areata a. Mees line in arsenic poisoning
c. Telogen effluvium b. Pterygium of nails in lichen planus
d. Androgenic alopecia Ref: Neena Khanna 4/e p130 c. Oncholysis in psoriasis
34. Alopecia areata is treated by: (JIPMER 92, DNB 07) d. Koilonychia in megaloblastic anemia (B12 Def)
a. Minoxidil Ref: Neena Khanna 4/e p144
b. Tranquilizers 43. Pitting nail dystrophy seen in: (PGI June 08, DNB 09)
c. Whitfields ointment a. Dermatophytic infection (AIIMS 01, UPSC 04)
d. Parenternal penicillin Ref: KD Tripathi 7/e p567 b. Psoriasis (JIPMER 98, DNB 99)
35. Diagnosis of a man with diffuse hair loss involving crown c. Lichen planus
& frontal scalp with maintenance of frontal hair line: d. Seborrhic dermatitis Ref: Neena Khanna 4/e p142
(JIPMER 98, DNB 01) 44. Which of the following is wrong statement: (AI 00)
a. Alopecia areata a. Koilonychia in vit B12 deficiency
b. Anagen effluvium b. Oncholysis in psoriasis
c. Male pattern baldness c. Mees lines in arsenic poisoning
d. Female pattern baldness Ref: Neena Khanna 4/e p133 d. Pterygium of nails in lichen planus
36. Contraindicated in androgenic alopecia: Ref: Neena Khanna 4/e p143
a. Testosterone (AI 2K, DNB 10) 45. Koenen’s periungal fibroma is seen in: (PGI 96, JIPMER 02)
b. Minoxidil a. Tuberous sclerosis
c. Cyproterone b. Neurofibromatosis
d. Finasteride Ref: Neena Khanna 4/e p134-135 c. Psoriasis
37. Most common type of non scarring alopecia is: d. Alopecia areata Ref: Neena Khanna 4/e p32
a. Androgenic 46. A Patient presented with yellowish discoloration and
b. Areata thickening of nails. He also has tunneling of 2 toe and 1
c. Tinea capitis Finger nail. Diagnosis can be done by: (PGI Nov 11)
d. Traction Ref: Neena Khanna 4/e p132 a. Wood’s Lamp
38. Pitting of nails can be seen in: b. KOH. Mount
a. Tinea unguium c. Biopsy
b. Alopecia areata d. Trank Smear Ref: Neena Khanna 4/e p191
c. Androgenic alopecia
d. Peripheral vascular disease Ref: Neena Khanna 4/e p130
E. MISCELLANEOUS
D. DISORDERS OF NAILS 47. Regarding Fordyce spots what is true: (PGI 02)
a. Represent internal maliganancy
39. Onychomycosis associated with HIV infection is: b. Ectopic sebaceous glands
a. Total dystrophic onychomycosis (J & K 2012) c. Present in axillae
b. Superficial white onychomycosis d. Found in healthy people
c. Distal lateral olnychomycosis e. Are erythematous
DERMATOLOGY

d. Proximal subungual onychomycosis Ref: Neena Khanna 4/e p123


Ref: Harrison 18/e p1652 48. Fordyce’s disease mainly involves: (PGI 97, AIIMS 2K)
40. Nail is involved in: (PGI Dec 07) a. Lips
a. Psoriasis b. Buccal mucosa
b. Lichen planus c. Neck
c. Fungal infection d. Trunk
d. Alopecia 49. In Fox Fordyce disease, true is/are: (PGI 2002)
e. Viral infection Ref: Neena Khanna 4/e p141-143 a. Common in adult woman
41. Pterygium of nail is characteristically seen in: b. Bullous lesions are common
a. Lichen planus (AI 06, DNB 03) c. Common in areola & axilla
b. Psoriasis d. Associated with other malignancies Ref: Internet

Ans. 33. b. Alopecia areata 34. a. Minoxidil 35. d. Female pattern .... 36. a. Testosterone
37. a. Androgenic 38. b. Alopecia areata 39. b. Superficial white… 40. a, b and c
41. a. Lichen planus 42. d. Koilonychia in … 43. b. Psoriasis 44. a. Koilonychia in Vit......
45. a. Tuberous sclerosis 46. b. KOH. Mount 47. b and d 48. a and b
49. a and c
3. PIGMENTARY DISORDERS

A. General
B. Vitiligo
C. Pityriasis
D. Miscellaneous
130 Jaypee’s Triple A

PIGMENTARY DISORDERS (QUESTIONS)

A. GENERAL 8. True about vitiligo are all except: (JIPMER 04, WB 05)
a. Genetic predisposition is known
1. Hyperpigmented lesions are: (PGI Nov 11) b. Leucotrichia is associated with good prognosis
a. Pityriasis alba c. PUVA-B is used for treatment
b. Melanoma d. Topical steroids give good results
c. Naevus anaemicus Ref: Neena Khanna 4/e p150
d. Dyskeratosis congenital 9. In a patch of vitiligo: (AIIMS 04, PGI 94)
e. Lentigines lichen planus a. Melanin synthesis is inhibited
Ref: Harrison 18/e p412; Neena Khanna 4/e p156 b. Melanosomes are absent
2. Which of the following is/are not the cause of c. Melanocytes are absent
hypopigmentation: (PGI May 10) d. Melanocytes are reduced
a. Leprosy Ref: Neena Khanna 4/e p149-150
b. Pinta 10. Vitiligo vulgaris, treatment is: (Jharkhand 06, DNB 04)
c. Syphilis a. PUVA (Bihar 03, DNB 01)
d. Pityriasis alba b. Steroids
e. Pityriasis versicolor Ref: Harrison 18/e p410 c. Coaltar
3. Hypopigmentation is/are seen in: (PGI May 11) d. All Ref: Neena Khanna 4/e p151, 154, 155
a. Vitiligo
b. Pityriasis versicolor
c. Lichen planus C. PITYRIASIS
d. Melasma
11. Pityriasis alba may be a manifestation of: (J & K 2012)
e. Scleroderma Ref: Neena Khanna 4/e p147
a. Atopic dermatitis
4. A newborn child presents with solitary, white, well-defined b. Worm infestation
hypopigmented patch on his right thigh. Diagnosis is: c. Indeterminate leprosy
(AIIMS 2K) d. Vitiligo
a. Piebaldism Ref: Neena Khanna 4/e p267
b. Albinism
12. Pityriasis versicolor is caused by:
c. Nevus achromicus
A. Candida
d. Acral vitiligo Ref: Neena Khanna 4/e p153
B. Rhinosdporodium
5. A female has hypopigmented lesion on centre of forehead, C. Malsezzia
drug, responsible is: (AIIMS Nov 2008) D. Tinea Ref: Neena Khanna 4/e p290
a. Hydroquinone
13. Pityriasis rosea true: (AI 07, AIIMS Nov 06)
b. Monobenzene metabolite of hydroquinone
a. Self limiting
c. Para tetra butyl catechol
b. Chronic relapsing
d. Para tetra butyl phenol Ref: Neena Khanna 4/e p145-152
c. Life threatening infection (autoimmune disease)
d. Caused by dermatophytes
DERMATOLOGY

B. VITILIGO Ref: Neena Khanna 4/e p53


14. Which viral association is found in pityriasis rosea:
6. Psoralen-A is used in the treatment of: (Delhi 2009 Feb) a. HHV 7 (Jharkhand 2004)
a. Pemphigus b. CMV
b. Vitiligo c. Vericella Zoster
c. Pityriasis alba d. EBV
d. Icthyosis Ref: Neena Khanna 4/e p154 Ref: Neena Khanna 4/e p53
7. Vitiligo is seen in: (COMED 2010) 15. Annular herald (mother) patch is seen in: (PGI 03, 96)
a. Blepharitis a. Psoriasis (TN, Kerala 98, AIIMS 97, 91, 90)
b. Vogt koyanagi harada syndrome b. P. alba
c. Syphilitic uveitis c. P. rosea
d. Posner-schlossman syndrome d. Nocardiasis Ref: Neena Khanna 4/e p54
Ref: Harrison 18/e p410

Ans. 1. b, d and e 2. b. Pinta 3. a and b 4. c. Nevus achromicus


5. d. Para tetra butyl... 6. b. Vitiligo 7. b. Vogt Koyanagi... 8. b. Leucotrichia is...
9. c. Melanocytes are... 10. a. PUVA 11. a. Atopic dermatitis 12. c. Malsezzia
13. a. Self limiting 14. a. HHV 7 15. c. P. rosea
Pigmentary Disorders 131
16. A 16-year-old boy presented with asymptomatic, multiple his classmates also have similar lesions. The most probable
erythamatous annular lesions with a collarette of scales at diagnosis is:
periphery of the lesions present on the trunk. The most a. Pityriasis rosea (AIIMS 01, 2K, 99, 98, 95)
likely diagnosis is: (AI 12, 05) b. Pityriasis versicolor
a. Pityriasis versicolor c. Indeterminate leprosy
b. Pityriasis alba d. Pityriasis alba Ref: Neena Khanna 4/e p267
c. Pityriasis rosacea 24. A 5 year boy has recurrent multiple asymptomatic oval and
d. Pityriasis rubra pilaris Ref: Neena Khanna 4/e p53-55 circular faintly hypopigmented macules with fine scaling
17. A patient of hypertension on ACE inhibitors developed on his face. The most probable clinical diagnosis is:
rosea skin erruptions. True statement regarding this a. Pityriasis versicolor (AI 03, AIIMS 96)
situation is: (SGPGI 2003) b. Indeterminate leprosy
a. Drug may be the cause and discontinuation may improve c. Pityriasis alba
the skin condition d. Acrofacial vitiligo Ref: Neena Khanna 4/e p267
b. High dose steroids are needed initially 25. True about pitryiasis alba: (PGI 01)
c. ACE inhibitors are safe and cannot lead to skin erruptions a. No active treatment required
d. Drug may be the cause discontinuation is not required b. Common in elderly
Ref: Harrison 18/e p400 c. Variant of vitiligo
18. Tinea Versicolor is caused by: (AI 05, 02, PGI 98) d. Common over the face
a. E. flaccosum e. Presents as scaly, whitish macules
b. Malassezia furfur Ref: Neena Khanna 4/e p267
c. T. rubrum 26. Exfoliative dermatitis is seen in all the following except:
d. T. schonleini Ref: Neena Khanna 4/e p291 a. Pityriasis rosea (AI 02, PGI June 06)
19. An adult presents with oval scaly hypopigmented macules b. Pityriasis rubra pilaris
over chest and back. The diagnosis is: c. Psoriasis
a. Leprosy (AIIMS 2001, AI 12) d. Drug hypersensitivity
b. Lupus vulgaris e. Eczema Ref: Neena Khanna 4/e p53
c. Pityriasis versicolor 27. Collarette of scales is a feature of:
d. Lichen Planus Ref: Neena Khanna 4/e p291 a. Acne
20. A 24-year-old man had multiple, small hypopigmented b. Psoriasis
macules on the upper chest and back for the last three c. Pityriasis rosacea
months. The macules were circular, arranged around d. Pemphigus Ref: Neena Khanna 4/e p53, 54
follicles and many had coalesced to form large sheets. The
surface of the macules showed fine scaling. He had similar
lesions one year ago which subsided with treatment. The D. MISCELLANEOUS
most appropriate investigation to confirm the diagnosis is;
a. Potassium hydroxide preparation of scales 28. Keratoderma is a feature of: (DP PGMEE 2009)
b. Slit skin smear from discrete macules a. Pityriasis rubra pilaris
c. Tzanck test (AIIMS May 12, Nov 03) b. Pemphigus
d. Skin biopsy of coalesced macules c. Pityriasis rosea
Ref: Neena Khanna 4/e p288 d. Psoriasis Ref: Neena Khanna 4/e p62; Roxburgh 18/e p153
29. Which of the following statements is true regarding

DERMATOLOGY
21. All of the following is given for the treatment for Pityriasis
versicolor except: (AI 05, 02) pityriasis rubra pilaris: (PGI 01)
a. Ketoconazole a. Isolated patches of normal skin are found
b. Griseofulvin b. Cephalocaudal distribution
c. Clotrimazole c. I.V. cyclosporine is effective and 1st line drug
d. Selenium sulphate Ref: Neena Khanna 4/e p292 d. More common in females
e. Methotrexate is effective Ref: Neena Khanna 4/e p62, 63
22. Griseofulvin is not useful in one of the following:
a. Tinea capitis (AI 02) 30. Woods lamp used in diagnosis of: (PGI Dec 06)
b. Tinea cruris a. P. versicolor
c. Pityriasis versicolor b. Vitiligo
d. Tinea pedis Ref: Neena Khanna 4/e p292 c. Porphyria
d. Psoriasis
23. Babloo around 10 year old boy presents with multiple small
e. Lichen Planus
hypopigmented scaly macules patches on cheek. Some of
Ref: Rook’s 8/e p. 5.19; Harrison 18/e p394

Ans. 16. c. Pityriasis rosacea 17. a. Drug may be the cause... 18. b. Malassezia Furfur 19. c. Pityriasis Versicolor
20. a. Potassium hydroxide 21. b. Griseofulvin 22. c. Tinea versicolor 23. d. Pityriasis alba
24. c. Ptyriasis alba 25. a, d and e 26. a. Pityriasis rosea 27. c. Pityriasis rosacea
28. a. Pityriasis rubra... 29. a, b and e 30. a, b and c
4. ALLERGIC DISORDERS AND
DERMATITIS (ECZEMA)

A. Dermatitis (Eczema)
B. Urticaria
C. Angioedema/Quincke Disease
D. Miscellaneous
Allergic Disorders and Dermatitis (Eczema) 133

ALLERGIC DISORDERS AND DERMATITIS (ECZEMA) (QUESTIONS)

A. DERMATITIS (ECZEMA) 8. Patch test is done to document:


a. Type I hypersensitivity
1. Most common site of atopic dermatitis is: (Delhi 2009 Feb) b. Delayed type hypersensitivity
a. Scalp c. Autoimmune disease
b. Elbow d. Immuno complex deposition Ref: Neena Khanna 4/e p16
c. Trunk 9. Worldwide most common cause for contact dermatitis is:
d. Anterior cubital fossa (AP 2011)
Ref: Harrison 18/e p395; Roxburgh 18/e p117 a. Nickel
2. The most potent topical corticosteroids is: (Delhi 2009 Feb) b. Chromium
a. Hydrocortisone butyrate cream 0.1% c. Gold
b. Betamethasone valerate cream 0.5% d. Mercury Ref: Roxburgh 18/e p132
c. Clobetasol propionate cream 0.5% 10. Which is specific for HIV and can be diagnosed by clinical
d. Clobetasone butyrate cream 0.5% examination? (AP 2010)
Ref: Internet a. Oral hairy leukoplakia
3. Seborrheic dermatitis is frequently seen in: b. Seborrheic dermatitis
(Delhi 2009 Feb) c. Norwegian scabies
a. Lipid storage disorders d. Reiter’s syndrome
b. Psoriasis vulgaris Ref: Neena Khanna 4/e p326
c. Hypertension 11. ‘Itch is disease’ is true for: (UPSC 2K)
d. Parkinson’s disease a. Atopic dermatitis
Ref: Harrison 18/e p398-406 b. Insect bites
4. The most common plant causing air-borne contact c. Seborrheic dermatitis
dermatitis is: (COMED 2010) d. Tinea cruris Ref: Neena Khanna 4/e p91
a. Semicarpus anacardium 12. Characteristic feature of atopic dermatitis is:
b. Parthenium hysterophorus a. Pruritus (MP 05, DNB 08)
c. Croton b. Dennie’s Lines
d. Primula obconica c. Scalling skin (Lichenification)
Ref: Neena Khanna 4/e p101 d. Rashes
5. Atopic dermatits is found to be associated with: Ref: Neena Khanna 4/e p91
(J & K 2010) 13. Minor clinical features in diagnosis of atopic dermatitis are
a. Increased IgA Levels all except: (PGI Dec 04)
b. Increased IgG levels a. Dry skin
c. Increased IgE levels b. Pruritus
d. Increased IgD levels c. Morgagnian folds
Ref: Neena Khanna 4/e p91 d. Pitriasis alba
6. Which of the following is an exogenous eczema? e. Dermographism

DERMATOLOGY
(J & K 2011) Ref: Neena Khanna 4/e p91, 94
a. Nummular (discoid) eczema 14. Dennie Morgan folds is seen in: (Karnataka 96, Bihar 06)
b. Seborrhoeic dermatitis a. Mastocytosis (Maha 2011)
c. Pityriasis alba b. Seborrhoic dermatitis
d. Eczematous polymorphic light eruption c. Sarcoidosis
Ref: Neena Khanna 4/e p194; Roxburgh 18/e p34 d. Atopic dermatitis
7. Which of the following tests is used for confirmation of Ref: Roxburgh 18/e p118
diagnosis of contact dermatitis? (DP PGMEE 2009) 15. Most common site of atopic dermatitis is:
a. Intradermal test a. Scalp (JIPMER 93, DNB 06)
b. Patch test b. Elbow
c. Scratch test c. Trunk
d. Prick test d. Anterior cubital fossa Ref: Neena Khanna 4/e p92
Ref: Neena Khanna 4/e p102

Ans. 1. d. Anterior cubital fossa 2. c. Clobetasol propionate... 3. d. Parkinson’s disease 4. b. Parthenium...


5. c. Increased IgE levels 6. d. Eczematous Polymorphic 7. b. Patch test 8. b. Delayed type...
9. a. Nickel 10. b. Seborrheic dermatitis 11. a. Atopic dermatitis 12. a. Pruritus
13. b. Pruritus 14. d. Atopic dermatitis 15. d. Anterior cubital fossa
134 Jaypee’s Triple A
16. Spongiosis is seen in: (Karnataka 03, AI 01) 23. Kaposi’s varicelliform eruption seen in:
a. Acute eczema a. Darrier disease (PGI 05, 04)
b. Lichen planus b. Varicella zoster
c. Psoriasis c. Pityriasis rosea
d. Pemphigus d. Atopic dermatitis
Ref: Neena Khanna 4/e p85, 86 e. Mumps
17. A 3-year-old child has eczematous dermatitis on extensor Ref: Neena Khanna 4/e p93
surfaces. His mother has a history of bronchial asthma. 24. After hepatitis B vaccination child with allergic family
Diagnosis could be: (AIIMS Nov 06, May 07, Nov 11) history and pruritis involving face & convexities developed
a. Atopic dermatitis numerous umblicated vesicles; which became pustular &
b. Contact dermatitis haemorhagic & crusted. After 2 days child developed high
c. Seborrhic dermatitis fever and lymphadenopathy. The diagnosis is: (AIIMS 07)
d. Infantile eczematous dermatitis a. Secondary infected atopic dermatitis
Ref: Neena Khanna 4/e p92 b. Molluscum contagiosum
18. An infant presented with erythematous lesions on cheek, c. Eczema herpaticum
extensor aspect of limbs, mother has history of bronchial d. Eczema vaccinatum
asthma, the probable diagnosis is: (AI 2007) Ref: Neena Khanna 4/e p93, 92
a. Air borne contact dermatitis 25. Most common cause of allergic contact dermatitis in Indian
b. Atopic dermatitis female is: (AIIMS 2000)
c. Seborrheic dermatitis a. Vegetables
d. Infectious eczematoid dermatitis b. Nail polish
Ref: Neena Khanna 4/e p92 c. Detergents
19. Rakesh, a 7-year-old boy had itchy, excoriated papules on d. Dyes
the forehead and the exposed parts of the arms and legs for Ref: Neena Khanna 4/e p101
3 years. The disease was most severe in the rainy season and 26. In India, the plant which causes dermatitis most commonly
improved completely in winter. Most likely diagnosis is: is: (AIIMS May 08)
(AIIMS 04) a. Parthenium grass
a. Insect bite hypersensitivity b. Cotton fibers
b. Scabies c. Poison ivy
c. Urticaria d. Ragweed
d. Atopic dermatitis Ref: Neena Khanna 4/e p102
Ref: Neena Khanna 4/e p91, 92 27. A 55-year-old male, with uncontrolled diabetes mellitus
20. A 25-year-old man presents with recurrent episodes of and hypertension, developed severe air-borne contact
flexural exzema, contact urticaria, recurrent skin infections dermatitis. The most appropriate drug for his treatment
and severe abdominal cramps and diarrhea upon taking sea would be: (AIIMS 04)
food. He is suffering from: a. Systemic corticosteroids
a. Seborrheic dermatitis (AI 04) b. Thalidomide
b. Atopic dermatitis c. Azathioprine
c. Airborne contact dermatitis d. Cyclosporine
d. Nummular dermatitis Ref: Neena Khanna 4/e p104
Ref: Neena Khanna 4/e p91, 92 28. Air–borne contact dermatitis can be diagnosed by:
DERMATOLOGY

21. Coin shaped eczema is: (Jharkhand 05) a. Skin biopsy (AIIMS May 06, DNB 10)
a. Nummular eczema b. Patch test
b. Atopic ecema c. Prick test
c. Infantile eczema d. Estimation of serum IgE levels
d. Endogenous eczema Ref: Neena Khanna 4/e p16
Ref: Roxburgh 18/e p126; 127; Harrison 18/e p377 29. Patch testing is done for: (PGI Dec 08)
22. Eczema herpeticum seen with: a. Atopic dermatitis
a. HSV (PGI June 07, 2K) b. Irritant contact dermatitis
b. EBV c. Allergic contact dermatitis
c. CMV d. Discoid eczema
d. VZV e. Seborrhoeic dermatitis
e. HPV Ref: Neena Khanna 4/e p280 Ref: Neena Khanna 4/e p16

Ans. 16. a. Acute eczema 17. a. Atopic dermatitis 18. b. Atopic dermatitis 19. d. Atopic dermatitis
20. b. Atopic dermatitis 21. a. Nummular eczema 22. a. HSV 23. d. Atopic dermatitis
24. c. Eczema herpaticum 25. c. Detergents 26. a. Parthenium grass 27. c. Azathioprine
28. b. Patch test 29. c. Allergic contact...
Allergic Disorders and Dermatitis (Eczema) 135
30. Increased level of IgE seen in: (DNB 2005) c. Cholinergic urticaria
a. Atopy d. Photodermatitis
b. Lepra reaction Ref: Neena Khanna 4/e p180
c. Cutaneous TB 39. A 9-year-old has multiple itchy erythematous wheals all
d. Lupus vulgaris Ref: Neena Khanna 4/e p91 over the body for 2 days. There is no respiratory difficulty.
31. Commonest site of atopic dermatitis is: Which is the best treatment? (AIIMS 04)
a. Scalp a. Antihelminthics
b. Elbow b. Systemic corticosteroids
c. Antecubital fossa c. Antihistamines
d. Trunk Ref: Neena Khanna 4/e p92 d. Adrenaline Ref: Neena Khanna 4/e p182
32. Perioral pallor and dennie’s line are seen in: 40. A 5-year-old male child has multiple hyperpigmented
a. Atopic dermatitis macules over the trunk. On rubbing the lesion with the
b. Chronic actinic dermatitis rounded end of a pen, he developed urticarial wheal,
c. Blood dyscrasia confined to the border of the lesion. The most likely
d. Perioral contact dermatitis diagnosis is: (AI 04)
Ref: Fitz Patricks Dermatology, 6/e p1180, 1181 1184, 1185, 1191 a. Fixed drug eruption
33. Most common cause of plant induced dermatitis in India: b. Lichen planus
a. Poison ivy c. Urticaria pigmentosa
b. Parathenium d. Urticarial vasculitis
c. Ragweed Ref: Roxburgh 18/e p214, 215
d. Cotton fibres Ref: Internet 41. Urticaria pigmentosa is a disorder related to:
a. Mast cells
b. Eosinophils
B. URTICARIA c. Neutrophils
d. Lymphocytes Ref: Neena Khanna 4/e p178
34. Following is a type of physical urticaria: (DP PGMEE 2009)
a. Dermographism
b. Urticaria pigmentosa C. ANGIOEDEMA/QUINCKE DISEASE
c. Urticaria vasculitis
d. Auto-immune urticaria Ref: Neena Khanna 4/e p179 42. Which one of the following statements is true for hereditary
35. Darrier’s sign is seen in: (DP PGMEE 2010) angioedema? (J & K 2010)
a. Xeroderma pigmentosa a. Hereditary Angioedema has a autosomal dominant
b. Urticaria pigmentosa inheritance pattern and occurs in about 1:150000
c. Herpes zoster population
d. Glucagouoma Ref: Roxburgh 18/e p214 b. Hereditary Angioedema is an autosomal recessive
disorder and occurs in about 1: 10000 population
36. Most common cause of physical urticaria is: (AP 2010)
c. Hereditary Angioedema is a sex linked disorder with a
a. Dermographism
female to male ratio of 2:1
b. Solar urticaria
d. Hereditary Angioedema is actually a misnomer and
c. Cholinergic urticaria
often found to occur sporadically in population
d. Pressure urticaria Ref: Neena Khanna 4/e p179
Ref: Neena Khanna 4/e p181

DERMATOLOGY
37. A 22-year-old woman developed small itchy wheals after
43. The following agents are found helpful in treatment of C1
physical exertion, walking in the sun, eating hot spicy food
esterase deficiency induced angioedema except: (J & K 2010)
and when she was angry. The most likely diagnosis is:
a. C1 esterase concentrate
(AIIMS Nov 03)
b. Danazol
a. Chronic idiopathic utricaria
c. Tranexemic acid
b. Heat urticaria
d. Epinephrine Ref: Neena Khanna 4/e p181
c. Solar urticaria
d. Cholinergic urticaria Ref: Neena Khanna 4/e p180 44. A person present with recurrent swelling on face and lips
due to emotional stress, cause is: (AIIMS May 2009)
38. A patient gets reccurent urticaria while doing exercise and
a. C1 esterase inhibitor deficiency
on exposure to sunlight. Which of the following is most like
b. Allergy
cause: (AIIMS 2000)
c. Anaphylaxis
a. Chronic idiopathic urticaria
d. None of the above
b. Universal dermographism
Ref: Neena Khanna 4/e p181

Ans. 30. a. Atopy 31. c. Anticubital... 32. a. Atopic dermatitis 33. b. Parathenium
34. a. Dermographism 35. b. Urticaria pigmentosa 36. a. Dermographism 37. d. Cholinergic urticaria
38. c. Cholinergic Utricaria 39. c. Antihistamines 40. c. Urticaria pigmentosa 41. a. Mast cells
42. a. Autosomal dominant 43. d. Epinephrine 44. a. C1 esterase...
136 Jaypee’s Triple A
45. A patient presents with history of episodic painful edema
of face and larynx and abdominal pain associated with D. MISCELLANEOUS
stress. Which of the following is likely to be deficient:
a. Complement C3 (AI 2009, May 10) 50. A 27-year-old male has itchy, excoriated papules on forehead
b. Complement C5 and exposed parts of arms and legs for 3 years. The disease
c. C1 esterase inhibitor was most severe in rainy season and improved completely
d. Properidin in winters. Most likely diagnosis is:
Ref: Neena Khanna 4/e p181 a. Scabies (AIIMS May 12)
b. Urticaria
46. Immediately after eating, a man develops swelling of face
c. Atopic dermatitis
and lips, respiratory distress, intense pruiritis, hypotension
d. Insect bite hypersensitivity
and feeling of impending doom. The most likely diagnosis
Ref: Neena Khanna 4/e p336, 337
is: (AI 2009)
a. Angioneurotic edema 51. Patch test is a type of: (AIIMS May 2009)
b. Anaphylaxis a. Immediate hypersensitivity
c. Myocardial infarction b. Antibody mediated hypersensitivity
d. Food stuck in throat c. Immune complex mediate hypersensitivity
Ref: Roxburgh 18/e p95 d. Delayed type hypersensitivity
Ref: Neena Khanna 4/e p16
47. Laboratory evaluation of a patient with recurrent lip edema
shows decreased C4 and C1INH (quantity & function) with 52. Skin test can be done for which hypersensitivity reactions:
normal C1q. Diagnosis is: (PGI 01)
a. Hereditary angioedema type II a. I
b. Hereditary angioedema type I b. II
c. Acquired AE type II c. III
d. Acquired AE type I d. IV
Ref: Neena Khanna 4/e p163-167
48. Not true about angioneurotic edema? (AI 2009)
a. Pitting edema of face, lips and mucous membrane 53. All are true/ except regarding patch test: (PGI 08)
b. C1 Esterase inhibitor deficiency can cause it a. Diagnose ABCD
c. Extreme temperature exposure can provoke it b. Read after 48 hours
d. Known with ACE inhibitors c. Angry back l/t false negative test
Ref: Neena Khanna 4/e p181 d. Reading is delayed in neomycin
e. T.R.U.E test
49. A man takes peanut and develops, tongue swelling, neck
Ref: Neena Khanna 4/e p16; Harrison 18/e 396; Roxburgh 18/e p133, 134
swelling, stridor, hoarseness of voice. What is the probable
diagnosis. (AIIMS Nov 06) 54. Morbilliform eruptions is seen in: (PGI 01)
a. Angioneurotic edema a. Scarlet fever
b. FB bronchus b. Rubella
c. Parapharyngeal abscess c. Toxic shock syndrome
d. FB in larynx d. Measles
Ref: Neena Khanna 4/e p181 e. Mumps
Ref: Neena Khanna 4/e p282
DERMATOLOGY

Ans. 45. c. C1 Esterase Inhibitor 46. b. Anaphylaxis 47. b. Hereditary angioedema... 48. a. Pitting edema of...
49. a. Angioneurotic... 50. d. Insect bite... 51. d. Delayed type... 52. a. I
53. c. Angry back... 54. c and d
5. PAPULOSQUAMOUS DISORDERS

A. Psoriasis
B. Lichen Planus
C. Miscellaneous
138 Jaypee’s Triple A

PAPULOSQUAMOUS DISORDERS (QUESTIONS)

A. PSORIASIS 9. About micro-munro abscesses which of the following


statements are true: (PGI June 09)
1. Psoriasis is characterized by all, except: (Delhi 2009 Feb) a. Seen in stratus corneum
a. Definite pink plaque with clear margin b. Seen in psoriasis
b. In children disappear in 2 weeks to reappear again c. Contain neutrophils and lymphocyte
c. Always associated with nail infection d. Contain neutrophils only
d. Involves knee and elbow e. Associated pustules are normally seen
Ref: Neena Khanna 4/e p41-43 Ref: Neena Khanna 4/e p48
2. Pitting of nails is seen in: (Delhi 2009 Feb) 10. HPR finding in psoriasis: (PGI June 09, Dec 07, 06)
a. Lichen planus a. Micromunro abscess
b. Psoriasis b. Suprapapillary thining
c. Pemphigus c. Grenz zone present
d. Arsenic poisoning Ref: Roxburgh 18/e p138, 139 d. Pautrier’s abscess
3. Which one of these should not to be used in severe e. Hyperkeratosis Ref: Neena Khanna 4/e p47, 48
widespread psoriasis? (COMED 2010) 11. Bleeding spots seen on removal of scales in psoriasis is
a. Methotrexate called as: (PGI June 2008)
b. Oral retinoids a. Auspitz sign
c. Cyclosporine b. Punctuate hemorrhage
d. Oral glucocorticoids Ref: Roxburgh 18/e p400 c. Nikolyski’s sign
4. Which of the following is the most common type of d. Darrier sign Ref: Neena Khanna 4/e p42
psoriasis? (Karnataka 2010) 12. A patient presents with erythematous scaly lesions on
a. Guttate psoriasis extensor aspect of elbows and knee. The clinical diagnosis
b. Pustular psoriasis is got by: (AIIMS May 02)
c. Stable plaque psoriasis a. Auspitz sign
d. Erythrodermic psoriasis Ref: Harrison 18/e p398 b. KOH smear
5. This type of psoriasis is commonly seen in children and c. Tzanck smear
may follow a streptococcal sore throat: d. Skin biopsy Ref: Neena Khanna 4/e p42
(Karnataka 2011) 13. A 30-years-old male presented with silvery scales on elbow
a. Guttate and knee, that bleed on removal. The probable diagnosis is:
b. Stable plaque a. Pityriasis (AI 08)
c. Pustular b. Seborrhoeic dermatitis
d. Arthropathic c. Psoriasis
Ref: Harrison 18/e p398; Neena Khanna 4/e p43 d. Secondary syphilis Ref: Neena Khanna 4/e p40, 41
6. Psoriasis is a: (J and K 2011) 14. Bulkeley membrane is seen in: (DNB 99, PGI 02)
a. B lymphocyte mediated disorder a. Psoriasis
b. T lymphocyte mediated disorder b. Pemphigus
c. Basophil mediate disorder c. Tinea
DERMATOLOGY

d. Eosinophil mediated disorder d. Pityriasis Ref: Neena Khanna 4/e p42


Ref: Nenna Khanna 4/e p41t 15. The important feature of psoriasis is: (Bihar 04, AMC 99)
7. Most common cause of psoriasis: (Kerala PG 09) a. Crusting
a. Autoimmune b. Scaling
b. Hereditary c. Oozing
c. Psychosomatic d. Erythema Ref: Neena Khanna 4/e p40, 41
d. Infective Ref: Neena Khanna 4/e p41 16. All of the following are seen in psoriasis except:
8. Auspitz sign is feature of: (Maha 2011) a. Auspitz sign present (SGPGI 2003)
a. Psoriasis b. 10% associated with arthritis
b. Lichen planus c. It is premalignant disease
c. Pityriasis alba d. Worsening of disease during winter
d. All of the above Ref: Neena Khanna 4/e p42 Ref: Neena Khanna 4/e p40-52

Ans. 1. c. Always associated... 2. b. Psoriasis 3. b. Oral retinoids 4. c. Stable plaque...


5. a. Guttate 6. b. T lymphocyte... 7. a. Autoimmune 8. a. Psoriasis
9. a, b and d 10. a, b and e 11. a. Auspitz sign 12. a. Ausptiz sign
13. c. Psoriasis 14. a. Psoriasis 15. b. Scaling 16. c. It is premalignant disease
Papulosquamous Disorders 139
17. All are true about psoriasis except: (PGI 2000) 26. The only indication of giving corticosteroids in pustular
a. Very pruritic psoriasis is: (AI 2005)
b. Pitting of nails a. Psoriatic erythroderma with pregnancy
c. Joint involvement in 5-10% b. Psoriasis in a pt. with alchoholic cirrhosis
d. Parakeratosis and acanthosis c. Moderate arthritis
e. Munro abscess Ref: Neena Khanna 4/e p40-43 d. Extensive lesions Ref: Neena Khanna 4/e p53
18. All are true regarding Psoriasis except: (AI 2000) 27. DOC for a pregnant woman in 2nd trimester with pustular
a. Arthritis in 5% psoriasis is:
b. Abscess is seen a. Prednisolone (AI 08)
c. Head, neck and face are not involved b. Dapsone
d. No scaly, red lesions are seen in inflamammary and natal c. Acitretin
area Ref: Neena Khanna 4/e p40-45 d. Methotrexate
19. Least common site involvement in psoriasis is: 28. Only definitive indication of systemic steroids in psoriasis
a. Scalp involvement (AI 98, PGI 02) is: (AIIMS Nov 11)
b. Nail Involvement a. Pustular psoriasis
c. CNS involvement b. Erythroderma
d. Arthritis Ref: Neena Khanna 4/e p40-53 c. Psoriatic arthropathy
20. Vitamin D analogue calcitriol is useful in the treatment of: d. Impetigo herpetiformis
a. Lichen Planus (AI 94, UP 04, Jhar 05) Ref: Neena Khanna 4/e p52, 53
b. Psoriasis 29. Pseudokoebner’s phenomenon is/are seen in:
c. Phemphigus a. Warts (PGI Nov 10, AI 11)
d. Leprosy Ref: Neena Khanna 4/e p50 b. Molluscum contagiosum
21. The treatment of choice for erythrodermic psoriasis is: c. Lichen planus
a. Corticosteriods (SGPGI 04) d. Psoriasis
b. Methotrexate e. Vitiligo Ref: Neena Khanna 4/e p41
c. Coal tar topically 30. Subepidermal lesion are: (PGI June 09)
d. Topical corticosteroids Ref: Neena Khanna 4/e p51 a. Bullous pemphigoid
22. Treatment of pustular psoriasis is: b. Pemphigus vulgaris
a. Thalidomide (AIIMS May 02, PGI 03) c. Hailey-Hailey disease
b. Retinoids d. Darier’s disease
c. Hydroxyurea e. Acanthosis nigricans Ref: Neena Khanna 4/e p77
d. Metholtrexate Ref: Neena Khanna 4/e p51 31. Photochemotherapy is useful in: (DNB 2005)
23. A patient with psoriasis was started on systemic steroids. a. Pityriasis rosea
After stoping the treatment, patient developed universally b. Psoriasis
red scaly skin with plaques losing their margins all over his c. Lichen planus
body. The most likely cause is: (AIIMS 01) d. Ichthyosis vulgaris Ref: Neena Khanna 4/e p52
a. Drug induced reaction 32. All are seen in Reiter’s syndrome except:
b. Pustular psoriasis a. Subcutaneous nodules
c. Bacterial infection b. Oral ulcers
d. Erythrodermic psoriasis Ref: Neena Khanna 4/e p46 c. Keratoderma blenorrhagicum

DERMATOLOGY
24. A patient with psoriasis was started on systemic steroids. d. Circinate balanitis Ref: Neena Khanna 4/e p43
After stopping treatment, the patient developed generalized 33. Berkeley membrane is seen in:
pustules all over the body. The cause is most likely to be: a. Psoriasis
a. Drug induced reaction (AI 02) b. Pemphigus
b. Pustular psoriasis c. Tinea capitis
c. Bacterial infections d. Pityriasis rubra
d. Septicemia Ref: Neena Khanna 4/e p46 Ref: Fitz Patricks Dermatology, 6/e p409, 418)
25. Treatment of erythematous skin rash with multiple pus 34. All are seen in Reiter’s syndrome except:
lakes in a pregnant woman is: (AI 2010) a. Subcutaneous nodules
a. Corticosteroids b. Oral ulcers
b. Retinoids c. Keratoderma blenorrhagicum
c. Methotrexate d. Circinate balanitis
d. Psoralen with PUVA Ref: Neena Khanna 4/e p53 Ref: Neena Khanna 4/e p43

Ans. 17. a. Very pruritic 18. c. Head, neck and face 19. c. CNS involvement 20. b. Psoriasis
21. b. Methotrexate 22. b. Retinoids 23. d. Erythrodermic Psoriasis 24. b. Pustular psoriasis
25. a. Corticosteroids 26. a. Psoriatic erythroderma 27. a. Prednisolone 28. d. Impetigo herpetiformis
29. a and b 30. a. Bullous pemphigoid 31. b. Psoriasis 32. a. Subcutaneous...
33. a. Psoriasis 34. a. Subcutaneous nodules
140 Jaypee’s Triple A
35. Treatment of psoriasis: (PGI June 05) 44. Wickhams stria is characteristic of: (Raj 2009)
a. PUVA a. Lichen planus
b. Methotrexate b. Psoriasis
c. Systemic steroids c. Dermatomyositis
d. Femicycline d. Pemphigoid Ref: Neena Khanna 4/e p57
e. Terbinafine Ref: Neena Khanna 4/e p50-52 45. A skin lesion which shows violaceous papules, polygonal
36. Which is not a complication of PUVA therapy: skin lesions, flat topped is seen in:
a. Premature aging of skin (AIIMS 97, PGI 2K) a. Lichen planus
b. Cataracts b. Psoriasis (Delhi 1995, Karnataka 2006)
c. Skin cancers c. Pityriasis rosea
d. Exfoliative Ref: Roxburgh 18/e p110, 327 d. Pityriasis rubra pilaris Ref: Neena Khanna 4/e p56
37. Photochemotherapy (Psoralent + UVV) is used in: 46. Mucosa is involved in: (PGI Dec 07)
a. Pityriasis rosea (SGPGI 03, AIIMS 95, AI 92, 91) a. Psoriasis
b. Erythroderma b. Lichen planus
c. Scabies c. Alopecia
d. Psoriasis Ref: Neena Khanna 4/e p54 d. Scabies
38. Uses of PUVA: (PGI Dec 04) e. Porphyria
a. Pigmented purpuric lesion Ref: Neena Khanna 4/e p57; Roxburgh 18/e p154-157
b. Herpes zoster 47. Features of lichen planus are: (PGI May 11)
c. Mycosis fungoides a. Pruritis
d. Lupus panniculitis b. Purple color
e. Lichenoid dermatitits of gougerot and blum c. Papule
Ref: Roxburgh 18/e p327 d. Purpura
39. Circulating lymphocytes are most sensitive to: e. Petechiae Ref: Neena Khanna 4/e p56
a. UV-A (UP 02, AIIMS 94) 48. True about lichen planus: (PGI Dec 04)
b. UV-B a. Basal cell degeneration
c. UV-C b. Colloid bodies seen
d. 760-800 mm c. Epidermal hyperplasia in chronic cases
40. The most effective treatment of pruritus in uremia is: d. Wickham’s striae seen
a. Ultraviolet light (AIIMS 95, PGI 02) e. Autoimmune disease
b. Cholestyramine Ref: Neena Khanna 4/e p56-60
c. Eskazine 49. Basal cell degeneration characteristically seen in:
d. Topical benzocaine Ref: Harrison 18/e p2311, 3212 a. Lichen planus (JIPMER 02, 98, PGI 02)
b. Psoriasis
c. Pemphigus
B. LICHEN PLANUS d. DLE
Ref: Neena Khanna 4/e p60
41. Polygonal voilaceous pruritic lesionis seen in:
50. A young lady presents with lacy lesions in oral cavity and
a. Pityriasis rosacea (Delhi 2009 Feb)
genitals, and her proximal nail fold has extended onto the
b. Lichen planus
nail bed. What is the likely diagnosis: (AI 12, 10)
c. Psoriasis
a. Psoriasis
DERMATOLOGY

d. Pemphigus vulgaris Ref: Neena Khanna 4/e p56


b. Geographic tongue
42. The most characteristic finding in lichen planus is: c. Lichen planus
a. Civatte bodies (DP PGMEE 2010) d. Candidiasis Ref: Neena Khanna 4/e p56, 58, 59
b. Basal cell degeneration
51. A 30-year-old male present with pruritic flat-topped
c. Thinning of nail plate
polygonal, shiny violaceous papules with flexural
d. Violaceous lesions Ref: Neena Khanna 4/e p608
distribution. the most likely diagnosis is:
43. Max Joseph space is a histopathological feature of? a. Psoriasis (Bihar 05, J and K 06)
a. Psoriasis (Maha 2011) b. Pityriasis
b. Lichen planus c. Lichen planus
c. Pityriasis rosea d. Lichenoid dermatitis
d. Parapsoriasis Ref: Neena Khanna 4/e p60 Ref: Neena Khanna 4/e p59

Ans. 35. a and b 36. d. Exfoliative 37. d. Psoriasis 38. a, c and e


39. c. UV-C 40. a. Ultraviolet light 41. b. Lichen planus 42. b. Basal cell...
43. b. Lichen planus 44. a. Lichen planus 45. a. Lichen planus 46. b. Lichen planus
47. a, b and c 48. a, b, c and d 49. a. Lichen planus 50. c. Lichen planus
51. c. Lichen planus
Papulosquamous Disorders 141
52. Lacy white lesion in mouth with pterygium is seen in:
a. Psoriasis (SGPGI 2001) C. MISCELLANEOUS
b. Pityriasis alba
c. Lichen planus 59. All are causes of papulosquamous lesions except:
d. Leprosy Ref: Neena Khanna 4/e p58 a. Psoriasis (PGI 02, 2K)
b. Parapsoriasis
53. Regarding lichen planus all are true, except: (AI 00)
c. Squamous cell carcinoma
a. Hypopigmentation in most residual disease
d. Mycosis fungoides
b. Lymphatic infiltration in supradermal layer
e. Congenital syphilis Ref: Neena Khanna 4/e p67
c. Itchy polygonal, purple papule
d. Skin, hair and oral mucosa commonly involved 60. All of the following may lead to plaque formation except:
Ref: Neena Khanna 4/e p56-89 a. Psoriasis (PGI 99, SGPGI 2003)
b. Lichen planus
54. All of the following regarding lichen planus are true
c. Pityriasis rosea
except: (MP 2005)
d. Pemphigus
a. Does not involve mucous membrane
Ref: Neena Khanna 4/e p39-71
b. Associated with Hepatitis ‘C’
c. Topical steroid are the mainstay of therapy a 61. Causes of erythroderma: (PGI June 05, 11)
d. Spontaneous remissions 6 months to 2 years a. Pityriasis alba
Ref: Neena Khanna 4/e p56-61 b. Pityriasis versicolor
c. Psoriasis
55. A patient presented with scarring alopecia, thinned nails,
d. Lichen planus
hypopigmented macular lesions over the trunk and oral
e. Eczema
mucosa. The diagnosis is: (AIIMS 01)
Ref: Neena Khanna 4/e p208, 40, 56, 85
a. Psoriasis
b. Leprosy 62. Necrotic keratinocytes occur in: (PGI Nov 11)
c. Lichen planus a. DLE
d. Pemphigus Ref: Neena Khanna 4/e p56-61 b. Graft versus host disease
c. Erythema multiformal
56. Characterstic nail finding in lichen planus:
d. Lichen planus
a. Pitting (AIIMS 01)
e. Psoriasis
b. Pterygium
Ref: Fitzpatrick’s 57/e p-242/44/51/63; Rooks 8/e p.41/47
c. Beau’s Lines
d. Hyperpigmentation of nails Ref: Neena Khanna 4/e p58 63. Which of the following are pruritic lesions:
a. Lichen planus (PGI 2000)
57. Itchy polygonal violaceous (itchy/prusitic) palpules seen
b. Sun burns
in: (AI 98, PGI 2K)
c. Pemphigoid
a. Psoriasis
d. Psoriasis
b. Pemphigus
e. SLE Ref: Neena Khanna 4/e p56, 41, 71, 77
c. Lichen planus
d. Pityriasis rosea Ref: Neena Khanna 4/e p59 64. Parakeratosis is defined as:
a. Retention of cytoplasmic contents in stratum corneum
58. Features of lichen planus are: (PGI May 11)
b. Retention of nuclei in stratum corneum
a. Pruritis
c. Increased thickness in stratum corneum
b. Purple
d. Decreased thickness of stratum corneum
c. Papule
Ref: Neena Khanna 4/e p47

DERMATOLOGY
d. Purpura
e. Petechiae Ref: Neena Khanna 4/e p59

Ans. 52. c. Lichen planus 53. a. Hypopigmentation... 54. a. Does not involve... 55. c. Lichen planus
56. b. Pterygium 57. c. Lichen planus 58. a, b and c 59. e. Congential syphilis
60. d. Pemphigus 61. c, d and e 62. a, b, c and d 63. a, b, c and d
64. c. Increased thickness..
6. PAPULOVESICULAR/
VESICOBULLOUS DISORDER AND
IMMUNOLOGICALLY-MEDIATED
DISEASES

A. General
B. Epidermolysis Bullosa
C. Pemphigus
D. Dermatitis Herpetiformis
E. Erythema Multiforme
F. Herpes
G. Epidermal Necrolysis, SJS, TEN and SSS
H. Miscellaneous
Papulovesicular/Vesicobullous Disorder & Immunologically-Mediated Diseases 143

PAPULOVESICULAR/VESICOBULLOUS... (QUESTIONS)

A. GENERAL B. EPIDERMOLYSIS BULLOSA


1. All are vesiculobullous lesions except: (Kerala 96, PGI 03) 9. In congenital dystrophic epidermolysis bullosa defect is
a. Dermatitis herpetiformis seen in: (AIIMS Nov 2008)
b. Scabies/atopic dermatitis a. Laminin 4
c. Pemphigus b. Collagen type 7
d. Pemphigoid Ref: Neena Khanna 4/e p91, 341 c. Collagen 4
2. Blister formation in burn case is in: d. Collagen 3 Ref: Harrison 18/e p3213
a. Intraepidermal (AI 2006) 10. In a 8-day-old child with no history of consanguinity in the
b. Subepidermal parents. The mother reports blisters and peeling of skin
c. Subdermal at the site of handling and pressure. There was a similar
d. Subfascial history in previous child which proved to be fatal. The
3. Intraepidermal blisters are seen in: (PGI Nov 2009) diagnosis:
a. Bullous pemphigoid a. Bullous pemphigod (AIIMS 01)
b. Pemphigus folliaceous b. Congenital syphillis
c. Dermatitis herpeteformis c. Congenital Epidermolysis bullosa
d. Bullous SLE d. Letterrer siwe disease Ref: Neena Khanna 4/e p30, 31
e. Bullous impetigo 11. A 2-day-old newborn girl born out of non-consanguinous
f. Pemphigus vulgaris marriage was evaluated for tense blister and areas of
g. Trauma (thermal) Ref: Roxburgh 18/e p104, 102 denuded skin that had been present since birth. The child
4. Acantholysis is seen in: (PGI May 10, June 09) develops their while mother handles her for bathing and
a. Bullous pemphigoid/SSS feeding. The sibling of child also had h/o developing
b. Dermatitis herpetiformis/Impetigo similar lesions diagnosis is: (AI 12)
c. Hailey- Hailey disease a. Congenital syphilis
d. Darrier’s disease b. Congenital epidermolysis bullosa
e. Pemphigus vulgaris Ref: Roxburgh 18/e p367, 266, 104 c. LCH
d. Congenital bullous icthyosiform erythroderma
5. Acantholysis involves (is seen in): (PGI Dec 08, AI 95)
Ref: Neena Khanna 4/e p30, 31
a. Epidermis
b. Dermis 12. A patient developed bullae without erythema on elbows,
c. Epidermis-dermis junction knee and sacral area f/b crust formation, scarring and milia.
d. Subcutaneous tissue He had no photosensitivity and negative family history for
e. Adipose tissue bullous diseases. On DIF IgG deposition at DEJ with no
f. All layers Ref: Neena Khanna 4/e p71 blood vessel involvement is seen probable diagnosis is:
a. EB dystrophic (AIIMS Feb 07)
6. Acantholytic cells are: (SGPGI – 2001)
b. EB acquisita
a. Epidermal cells
c. Porphyria CT
b. Plasma cells

DERMATOLOGY
d. Pemphigoid bullous
c. Keratinocytes
Ref: Neena Khanna 4/e p32 Roxburgh 18/e p102
d. Giant cells
7. Tzank smear helps in the diagnosis of:
a. Herpes viral infection (MP 05) C. PEMPHIGUS
b. Bullous pemphigoids
c. Carcinoma of cervix 13. Pruritus is seen in all, except:
d. None Ref: Neena Khanna 4/e p69, 74 (Delhi 2009 Feb)
8. Nikolsky sign is not present in: (Bihar 06, DNB 09) a. Pemphigus
a. Pemphigus b. Lichen simplex chronicus
b. Pemphigoid c. Psoriasis
c. Vitiligo d. Contact dermatitis
d. Staphylococcal scalded syndrome Ref: Neena Khanna 4/e p71, 105, 41, 100
Ref: Neena Khanna 4/e p73, 149

Ans. 1. b. Scabies/Atopic... 2. b. Subepidermal 3. b, e and f 4. c, d and e


5. a. Epidermis 6. c. Keratinocytes 7. a. Herpes viral infection 8. c. Vitiligo
9. b. Collagen type 7 10. c. Congential Epidermolysis 11. b. Congenital... 12. b. EB acquisita
13. a. Pemphigus
144 Jaypee’s Triple A
14. Pemphigus caused by exposure to black fly antigens is: 23. A patient with Bullous eruptions on lower limb and trunk,
a. Pemphigus vulgaris (J and K 2012) biopsy show epidermal bullae. The correct diagnosis is:
b. Endemic pemphigus foliaceus (AI 2000)
c. Pemphigus erythematosus a. Pemphigoid
d. IgA pemphigus b. Pemphigus vulgaris
Ref: Harrison 18/e p425 c. Impetigo
15. Intraepidermal blisters are seen in all of the following d. Internal malignany Ref: Neena Khanna 4/e p72, 73
conditions except: (DP PGMEE 2009) 24. A 24-years-old (middle aged) female has flaccid bullae in
a. Pemphigus foliaceous the skin and persistent painful oral erosions (palatal and
b. Pemphigus vulgaris vestibular lesions in buccal mucosa). Histopathology
c. Paraneoplastic pemphigus shows intraepidermal acantholytic blisters. The most likely
d. Pemphigoid diagnosis is:
Ref: Neena Khanna 4/e p71, 72 a. Bullous pemphigoid (PGI Dec 05, AI 08, 97)
16. A 25-year old male had pigmented macules over the palm, b. Erythema multiforme (AIIMS May 03)
sole and oral mucosa. He also had anemia and pain in c. Pemphigus vulgaris
abdomen. The most probable diagnosis is: d. Dermatitis herpetiformis
a. Albright’s syndrome (DP PGMEE 2010) e. Epidermolysis bullosa acquista
b. Cushing’s syndrome Ref: Neena Khanna 4/e p72, 73
c. Peutz-jegher’s syndrome 25. A 40-year-old male developed persistant oral ulcers followed
d. Incontinentia pigmenti by multiple flaccid bullae on trunk and extremities. Direct
Ref: Neena Khanna 4/e p158’ Roxburgh 18/e p313 examination of a skin biopsy immunoflurescence showed
17. Bullous skin lesions with ‘Intraepidermal split’ is feature of ? intercellular IgG deposits in the epidermis. The most
a. Pemphigus vulgaris (Maha 2011) probable diagnosis is: (AI 03)
b. Pemphigoid a. Pemphigus vulgaris
c. Epidermolysis bullosa b. Bullous Pemphigoid
d. All of the above Ref: Neena Khanna 4/e p71, 72 c. Bullous Lupus erythematosus
d. Epidermolysis bullosa acquisita
18. In pemphigus vulgaris, antibodies are present against:
Ref: Neena Khanna 4/e p72, 73; Roxburgh 18/e p102-104
a. Basement membrane (SGPGI 2000)
b. Intercellular substance 26. All are asociated with pemphigus except:
c. Keratin a. Thymoma (PGI 98, 02)
d. Cell nucleus Ref: Roxburgh 18/e p102 b. CLL
c. Myasthenia gravis
19. An autoimmune disease is: (AI 2000)
d. Non- Hodgkins lymphoma
a. Pemphigus vulgaris
e. Atrophic gastritis Ref: Harrison 18/e p426-428
b. Psoriasis
c. Lichen planus 27. Mucous lesions are seen in: (PGI 02, AIIMS 92)
d. Acne vulgaris Ref: Harrison 18/e p42 a. Sec. syphilis
b. Dermatitis herpetiformis
20. ‘Row of tombstones’ appearance is seen in:
c. Psoriasis
a. Irritant dermatitis (JIPMER 98, PGI 02)
d. Pemphigus
b. Pemphigus
e. Porphyria Ref: Neena Khanna 4/e p73
c. Pemphigoid
d. Herpes zoster 28. A 85-year-old woman with Nikolsky sign-ve, blisters on
DERMATOLOGY

thigh and trunk, lesions come on and off. What is the cause:
21. In pemphigus vulgaris, antibodies are present against:
a. Pemphigus vulgaris (PGI 2000)
a. Basement membrane (PGI 2000)
b. Pemphigoid
b. Intercellular substance
c. Lichen planus
c. Cell nucleus
d. Dermatitis herpetiformis
d. Keratin
e. Leprosy Ref: Neena Khanna 4/e p71-73
e. Cell membrane Ref: Neena Khanna 4/e p72, 73
29. “Intraepidermal” IgG deposition is seen in:
22. True about pemphigus vulgaris is all except: (PGI June 09)
a. Pemphigus
a. Subepidermal
b. Bullous pemphigoid
b. Autoimmune disease
c. Herpes genitalis
c. Tzanck smear shows acantholytic cells
d. SLE
d. Antibody are formed against desmogleins
Ref: Neena Khanna 4/e p71
e. Blister on skin and mucosa Ref: Roxburgh 18/e p102

Ans. 14. b. Endemic... 15. d. Pemphigoid 16. c. Peutz-Jegher’s... 17. a. Pemphigus vulgaris
18. b. Intercellular... 19. a. Pemphigus vulgaris 20. b. Pemphigus 21. b. Intercellular substance
22. a. Subepidermal 23. b. Pemphigus vulgaris 24. c. Pemphigus vulgaris 25. a. Pemphigus vulgaris
26. e. Atrophic gastritis 27. d. Pemphigus 28. b. Pemphigoid 29. a. Pemphigus
Papulovesicular/Vesicobullous Disorder & Immunologically-Mediated Diseases 145
37. Symmetrically distributed on the extremities with typical
D. DERMATITIS HERPETIFORMIS and often recurrent concentric “target” lesions are seen in:
a. Erythema nodosum (AP 2011)
30. All are true about dermatitis herpetiformis except: b. Erythema multiforme
a. More common in young adults c. Erythema infectosum
b. Intense pruritus (JHARKHAND 05) d. Erythema marginatum
c. Deposit of IgG at the epidermodermal lesion Ref: Neena Khanna 4/e p174
d. None Ref: Neena Khanna 4/e p80
38. Erythema infection is caused by: (AP 2010)
31. A 30-year-old male had severely itchy papulovesicular a. Echoviruses
lesions on both knees, elbows, upper back and buttocks for b. Parvoviruses
one year. Direct immunofluorescence staining of the lesions c. Paramyxovirus
showed IgA deposition at dermoepidermal junction and d. Coxsackie virus Ref: Harrison 18/e p1478
dermal papilla. The most probable diagnosis is:
39. Target lesions are seen in: (Kerala 2008)
a. Pemphigus vulgaris (AI 12, 04, AIIMS Nov 02)
a. Erythema marginatum
b. Bullous pemphigoid
b. Lichen planus
c. Dermatitis herpetiformis
c. psoriasis
d. Nummular eczema Ref: Neena Khanna 4/e p80
d. Erythema multiforme Ref: Neena Khanna 4/e p175
32. Extermely pruritic excoriation and papules on buttocks
40. Commonest etiology of erythema multiforme is:
with autoantibodies against epidermal transglutaminase
a. Viral (AI 95, DNB 99, UP 02)
and IgA deposition in dermis on immunohistological
b. Bacterial
examination of normal perilesional skin. Diagnosis is:
c. Food
a. Pemphigus vulgaris (SGPGI 01)
d. Drugs
b. Pemphigoid
Ref: Neena Khanna 4/e p174
c. Linear IgA disease
d. Dermatitis herpetiformis Ref: Neena Khanna 4/e p80 41. All are true about erythema multiformis except:
a. Due to herpes simplex (Jharkhand 05)
33. The treatment of dermatitis herpetiformis is:
b. Due to sulphonamide
a. Gluten free diet with minerals and vitamins
c. Lesion are symmetrical
b. Carbamazepine (AI 02, Kerala 96)
d. Mucous membrane is involved in all
c. Acyclovir
Ref: Neena Khanna 4/e p174, 175
d. Corticosteroids
Ref: Neena Khanna 4/e p82; Harrison 18/e p427; Roxburgh 18/e p100-101 42. Regarding erythema multiforme all are true except:
a. No vesicles (AI 2K)
34. What can patient with gluten sensitive hypersensitivity
b. Target lesions are seen
consume as food:
c. Involves face and neck regions
a. Rice (PGI June 2006)
d. Sign of internal malignancy
b. Barley
Ref: Neena Khanna 4/e p174-175; Harrison 18/e p152;
c. Oat
Roxburgh 18/e 82, 83
d. Corn
e. Rye Ref: Neena Khanna 4/e p82; Harrison 18/e p427, 428
35. All are true about linear IgA disease except: F. HERPES
a. Subepidermal involvement (PGI 2001)
b. Severe itching 43. Herpes zoster in commonly seen in: (Delhi 2009 Feb)

DERMATOLOGY
c. Granular deposition of IgA a. Cervical region
d. Are candidates for gluten free diet b. Thoracic region
e. A variant of dermatitis herpetiformis c. Lumbar region
Ref: Roxburgh 18/e p101; Harrison 18/e p428 d. Geniculate ganglion
Ref: Roxburgh 19/e p53, 54; Neena Khanna 4/e p277
44. A 45-year-old male has multiple grouped vesicular lesions
E. ERYTHEMA MULTIFORME present on the T10 segment dermatome associated with
pain. The most likely diagnosis is:
36. Erythema multiforme is most often related to:
a. Herpes zoster (AIIMS Nov 02)
a. Herpes zoster infection (DP PGMEE 2010)
b. Dermatitis herpetiformis
b. Herpes simplex infection
c. Herpes simplex
c. Molluscum contagiosum infection
d. Scabies
d. Human papilloma virus infection
Ref: Neena Khanna 4/e p277
Ref: Neena Khanna 4/e p280

Ans. 30. c. Deposit of IgG... 31. c. Dermatitis... 32. d. Dermatitis... 33. a. Gluten free diet...
34. a and d 35. c, d and e 36. b. Herpes simplex... 37. b. Erythema multiform
38. b. Parvoviruses 39. d. Erythema multiforme 40. a. Viral 41. d. Mucous membrane
42. a. No vesicles 43. b. Thoracic region 44. a. Herpes zoster
146 Jaypee’s Triple A
45. The most frequent cause of recurrent genital ulceration in a
sexually active male is: (AI 03) H. MISCELLANEOUS
a. Herpes genitalis
b. Aphthous ulcer 52. A young boy with multiple flaccid bullous lesions over
c. Syphilis trunk with oral mucosal lesions. Most likely finding on
d. Chancroid Ref: Roxburgh 18/e p52-54 biopsy would be: (AIIMS Nov 09)
a. ‘Fishnet’ IgG deposits in epidermis
46. Herpes resistant to acyclovir is treated by: (JIPMER 02)
b. Linear IgG in Deposits
a. Foscarnet
c. Linear IgA in dermal papillae
b. Lamivudine
d. Granular IgA in reticular dermis
c. Ganciclovir
Ref: Neena Khanna 4/e p72
d. Valocyclovir Ref: Fitzpatrick’s 7/e p1895, 2203-08
53. Inter cellular IgG deposition in epidermis is seen in:
47. Recurrent lesions on glans which heal with residual
a. Pemphigus (AIIMS May 09, Nov 11)
hyperpigmentation is suggestive of:
b. Sub corneal pustular dermatosis
a. Aphthous ulcer
c. Bullus pemphigoid
b. Fixed drug eruption
d. Dermatitis herpetiformis Ref: Neena Khanna 4/e p72
c. Herpes genitalis
d. Chlamydial Infection Ref: Neena Khanna 4/e p311 54. Direct immunofluorescence is positive in:
a. Atopic dermatitis (PGI 02)
b. SLE
G. EPIDERMAL NECROLYSIS, SJS, TEN c. Pemphigus
d. Secondary syphilis Ref: Neena Khanna 4/e p75, 225
AND SSS
55. A 40-year-old male had multiple blisters over the trunk and
48. Which drug cause Steven johnson’s syndrome: (Raj 2009) extremities. Direct Immunofluorescence studies showed
a. Cefadroxyl linear IgG deposits along the basement membrane, which
b. Penicillin of the following is the most likely diagnosis:
c. Ciprofloxacin a. Pemphigus vulgaris (AIIMS Nov 2004)
d. Azithromycin Ref: Neena Khanna 4/e p82 b. Bullous pemphigoid
c. Pemphigus foliaceous
49. A 60-year-old patient presented with several bullous
d. Dermatitis herpetiformis Ref: Neena Khanna 4/e p77-78
lesions for the last 3 days; each bulla was surrounded by
an erythematous halo. There were multiple target lesions. 56. Granular IgA deposit at dermal papilla are found in:
Patient also had oral erosions. The most likely diagnosis is: a. Dermatitis Herpetiformis (AIIMS May 09, Nov 11)
(SGPGI 2004) b. IgA disease of childhood
a. Chicken pox c. Herpetic gestation
b. Herpes simplex d. Bullous pemphigoid
c. Herpes zoster Ref: Neena Khanna 4/e p80; Harrison 18/e p427
d. Steven-johnson syndrome 57. Skin disease not showing DIF (Direct
Ref: Neena Khanna 4/e p175, 216, 414, 82 immunofluorescence): (PGI Nov 2010)
50. Toxic epidermonercrolysis is caused by: (PGI 04) a. Darrier’s disease
a. Phenytoin b. Hailey-Hailey disease
b. Penicillin c. Cicatricial pemphigoid
c. Erythromycin d. Dermatitis herpetiformis
DERMATOLOGY

d. Gold Ref: Neena Khanna 4/e p174 e. Pemphigus Ref: Neena Khanna 4/e p69-71
51. A 3-months-old male infant developed otitis media for 58. All of the following are true about bullous impetigo
which he was given a course of Co-trimoxazole. A few days except: (Karnataka 2010)
later, he developed extensive peeling of the skin; there were a. Caused by Staphylococcus aureus
no mucosal lesions and the baby was not toxic. The most b. Common in neonates and infants
likely diagnosis is: (AIIMS 04) c. Lesions commonly occur on face
a. Toxic epidermal necrolysis d. Lesions have an erythematous base
b. Staphylococcal scalded skin syndrome Ref: Neena Khanna 4/e p245
c. Steven johnsom syndrome 59. CREST syndrome, a limited scleroderma, consists of all, except:
d. Infantile pemphigus a. Raynaud’s disease (J and K 2010)
Ref: Neena Khanna 4/e p41, 84, 247; Roxburgh 18/e p245 b. Calcinosis and telangiectasia
c. Bullous skin eruptions
d. Esophageal dysmotity Ref: Harrison 4/e p2080

Ans. 45. a. Herpes genitalis 46. a. Foscarnet 47. c. Herpes Genitalis 48. b. Penicillamine
49. d. Steven Johnson... 50. a and b 51. b. Staphylococcal... 52. a. ‘Fishnet’ IgG...
53. a. Pemphigus 54. b and c 55. b. Bullous pemphigoid 56. a. Dermatitis...
57. a and b 58. d. Lesions have an... 59. c. Bullous skin eruptions
Papulovesicular/Vesicobullous Disorder & Immunologically-Mediated Diseases 147
60. Acanthosis nigricans histologically show: c. Psoriasis
(MHPGM-CET 2010) d. Boweli’s disease
a. Papillomatosis e. Hailey-hailey disease Ref: Roxburgh 18/e p266
b. Marked acanthosis 63. Spontaneous remission is most frequent with:
c. Hypermelanosis a. Herpes labialis (SGPGI 04, DNB 05)
d. All of the above b. Herpes genitalis
Ref: Robbins and Cotran Pathologic Basis of c. Herpetic chancroid
Disease 7/e p335 Table 7-12 d. Herpes gestationis Ref: Neena Khanna 4/e p82
61. Hailey – hailey disease is: (Jharkhand 03) 64. Commonest site of herpes gestationis is:
a. Benign familial chronic pemphigus a. Periumbilical region (AIIMS 93, Delhi 02)
b. Pemphigus acutus b. Flanks of abdomen
c. Pemphigus c. Vulva
d. Lyell’s syndrome Ref: Roxburgh 18/e p266 d. Infraorbital Ref: Neena Khanna 4/e p82
62. Dyskeratosis is characteristic feature of: (PGI 2000)
a. Darrier’s disease
b. Pemphigus vulgaris

DERMATOLOGY

Ans. 60. a. Papillomatosis 61. a. Benign familial... 62. a, d and e 63. d. Herpes gestationis
64. a. Periumbilical...
7. SYSTEMIC DISEASES
WITH PROMINENT CUTANEOUS
FEATURES, PHOTOSENSITIVITY
DISORDERS AND CUTANEOUS
DRUG ERUPTIONS

A. Autoimmune Systemic Diseases


B. Photosensitivity Disorders
C. Purpura
D. Porphyria
E. Neutrophilic Dermatoses
F. Panniculitis
G. Cutaneous Drug Eruptions
Systemic Diseases with Prominent Cutaneous Features... 149

SYSTEMIC DISEASES WITH PROMINENT... (QUESTIONS)

A. AUTOIMMUNE SYSTEMIC 9. Which of the following not a feature of dermatomyositis?


(AIIMS Nov 09)
DISEASES a. Gottron’s papules
b. Periungual telangiectasia
1. Chloroquine is used in the treatment of: (Delhi 2009 Feb)
c. Salmon rash
a. DLE
d. Mechanic’s hand
b. Pemphigus
Ref: Neena Khanna 4/e p227-229, 372
c. Psoriasis
d. Nummular eczema Ref: Roxburgh 18/e p87 10. True about dermatomyositis: (PGI Nov 09)
a. Gottron papules
2. Apple jelly nodules are seen in: (Delhi 2009 Feb)
b. ANA a/w all cases
a. Tuberculosis verrucosa cutis
c. All cases a/w malignancy
b. Tuberculous gumma
d. Proximal muscle wasting
c. Scrofuloderma
Ref: Neena Khanna 4/e p227, 228
d. Lupus vulgaris Ref: Neena Khanna 4/e p221
11. A 40-year-old woman presented with a 8 month history of
3. One of the following is an autosomal dominant systemic
erythema and swelling of the periorbital region & papules
amyloidosis: (J & K 2012)
& plaques on the dorsolateral aspect of forearms & knuckles
a. Familial mediterranean fever
with ragged cuticles. There was no muscle weakness. The
b. Muckle wells syndrome
most likely diagnosis is:
c. Lichen amyloidosis
a. SLE (AIIMS Nov 04, DNB 11)
d. Primary cutaneous amyloidosis
b. Dermatomyositis
Ref: Harrison 18/e p265-160
c. Systemic sclerosis
4. Heliotrope erythema is seen in: (J & K 2010) d. Mixed connective tissue disorder
a. Dermatomyositis Ref: Neena Khanna 4/e 228, 229; Harrison 18/e p3510
b. Drug induced rash
12. Antibody that is strongly associated with polymyositis:
c. Rheumatoid arthritis
a. Anti-jo1 (AIIMS Nov 08)
d. Leprosy Ref: Neena Khanna 4/e p228
b. Anti-ku
5. Cutaneous marker of diabetes mellitus: (Kerala PG 09) c. Anti-Scl-70
a. Candidal balanoposthitis d. Anti-sm Ref: Harrison 18/e p3510, 3511
b. Albinism
13. Female presents with history of color change from pallor
c. Necrobiosis lipoidica
to cyanosis on exposure to cold in fingers. This condition is
d. Lichen planus Ref: Neena Khanna 4/e p383
mostly associated with: (AIIMS Nov 08, DNB 05)
6. All of the hematological disorders cause pruritus, except: a. Scleroderma
(DP PGMEE 2010) b. Leukemia
a. Polycythemia rubra vera c. Lung infections
b. Hemolytic anemia d. Hepatosplenomegaly
c. Multiple myeloma Ref: Harrison 18/e p2762
d. Non-Hodgkin’s lymphoma

DERMATOLOGY
14. True about drug induced SLE is: (PGI 2K, DNB 05)
Ref: Harrison 18/e p937-942
a. CNS manifestation are common
7. Steroid is not given in: (Raj 2008, 2009) b. Renal involvement is common
a. Pemphigus c. Antihistone antibodies are found in many
b. Lichen planus d. All with antibodies progress to lupus
c. Psoriasis e. Sex ratio is nearly equal
d. Dermatomyositis Ref: Neena Khanna 4/e p71, 56 Ref: Harrison 18/e p2735
8. Periungual telagiectasia not seen in: (AP 2011) 15. Lupus like picture is causes by all except:
a. SLE a. Chloroquine (AI 91, DNB 03)
b. Dermatomyositis b. Procanamide
c. Scleroderma c. Hydralazine
d. Mixed connective tissue disorder d. Isoniazid Ref: Neena Khanna 4/e p224
Ref: Neena Khanna 4/e p237, 233

Ans. 1. a. DLE 2. d. Lupus vulgaris 3. b. Muckle wells syndrome 4. a. Dermatomyositis


5. c. Necrobiosis... 6. c. Multiple myeloma 7. d. Dermatomyositis 8. d. Mixed connective...
9. c. Salmon rash 10. a. Gottron papules 11. b. Dermatomyositis 12. a. Anti-jo1
13. a. Scleroderma 14. c. Antihistone antibodies 15. a. Chloroquine
150 Jaypee’s Triple A
16. Chloroquine is indicated in treatment of: c. Calcinosis cutis
a. Pemphigus (AI 92, DNB 10) d. Morphea
b. Pempigoid e. Prophyria cutanea tarda
c. Psoriasis Ref: Neena Khanna 4/e p224-388
d. DLE Ref: Neena Khanna 4/e p222 25. Which of the following is not photosensitive: (AI 04)
17. 23-year-old lady develops brown macular lesions over a. Porphyria
bridge of nose and cheek following exposure to light. The b. DLE
probable diagnosis is: (AIIMS May 12, 99) c. SLE
a. SLE d. Lichen planus Ref: Neena Khanna 4/e p56
b. Acne rosacea 26. Exposure to sunlight can precipitate:
c. Chloasma a. Chloasma (AI, UPSC 2K, DNB 02)
d. Photodermatitis Ref: Neena Khanna 4/e p156, 157 b. Discoid lupus erytyhematosus
18. A girl of 19 years with arthritis and photosensitive rash on c. Dermatitis herpetiformis
cheeks, likely diagnosis is: (AI 01, Bihar 03) d. Lupus vulgaris
a. SLE Ref: Harrison 18/e p430; 449-445; Neena Khanna 4/e p218
b. Chloasma 27. A 45-year-old farmer has itchy erythematous popular
c. Stevens johnson syndrome lesions on face, neck, ‘V’ area of chest, dorsum of hands and
d. Lyme’s disease Ref: Neena Khanna 4/e p224-226 forearms for 3 years. The lesions are more severe in summers
19. Commonest cutaneous eruption in systemic L.E. is: and improve by 75% in winters. The most appropriate test
(DNB 2005) to diagnose the condition would be:
a. Palmar erythema a. Skin biopsy (AI 06)
b. Discoid lesions b. Estimation of IgE levels in blood
c. Erythema of light exposed area c. Patch test
d. Diffuse morbiliform erythema d. Intradermal prick test Ref: Harrison 18/e p444, 445
Ref: Neena Khanna 4/e p122
20. Heliotrope rash is seen in:
a. SLE
C. PURPURA
b. Lichen Planus
28. Palpable purpura is seen in all except:
c. Pityriasis alba
a. Wegeners GN (PGI 99, DNB 07)
d. Dermatomyositis
b. ITP
Ref: Neena Khanna 4/e p227
c. HSP
21. Which of the following is not seen in Dermatomyositis: d. Serum sickness Ref: Harrison 18/e p421
a. Gottron papules
29. Palpable pupura is seen in all conditions except:
b. Helitrope rash
a. Cryoglobulinuria (AI 2K, DNB 06)
c. Mechanics hand
b. H. S. Pupura
d. Salmon rash Ref: Neena Khanna 4/e p227, 228
c. Giant cell arteritis
22. Commonest site of necrobiosis lipidoica diabeticorum is: d. Drug induced vasculitis
a. Face Ref: Harrison 18/e p421
b. Neck [Ref: Rook’s 8/e p. 49.1-49.51; 50.1-50.43;
c. Leg Fitzpatrick’s 7/e p-1379 – 83, 1639;
d. Arm Ref: Neena Khanna 4/e p383
DERMATOLOGY

30. A 42-year-old female has palpable purpura with rash over


buttocks, pain in abdomen, and arthropathy diagnosis is:
B. PHOTOSENSITIVITY DISORDERS a. Sweet syndrome (AI 08)
b. HSP
23. Photosensitivity is a feature of: (DP PGMEE 2009) c. Purpura fulminans
a. Porphyria cutanea tarda d. Meningococcemia
b. Psoriasis Ref: Harrison 18/e p421, 422
c. Pemphigoid 31. IgA deposits on skin biopsy: (AIIMS May 09)
d. Pompholyx Ref: Neena Khanna 4/e p387 a. Henoch schonlein purpura
24. Which of the following are photosenstive diseases: b. Giant cell arteritis
a. SLE (PGI 01, DNB 99) c. Microscopic polyangitis
b. Liver spots d. Wegener’s granulomatosis Ref: Harrison 18/e p422

Ans. 16. d. DLE 17. c. Chloasma 18. a. SLE 19. c. Erythema of light...
20. d. Dermatomyositis 21. d. Salmon rash 22. c. Leg 23. a. Porphyria...
24. a, b and e 25. d. Lichen planus 26. b. Discoid lupus... 27. a. Skin biopsy
28. b. ITP 29. c. Giant cell arteritis 30. b. HSP 31. a. Henoch...
Systemic Diseases with Prominent Cutaneous Features... 151
32. A 5-year-old child develops non blanching macules, papules 40. A patient gives h/o recurrent oral ulcers. The ulcers are
and petecheal hemorrhage on lower extremities, mild small with a yellow floor surrounded by an erythematous
abdominal pain, and skin biopsy showed IgA deposition halo on lips. He also has multiple, tender nodules on shin.
along blood vessels and perivascular neutrophilic infiltrate. The probable diagnosis is:
Most probably diagnosis is a. Pemphigus vulgaris (AIIMS Nov 11)
a. Wegner’s granulomatosis (AIIMS May 09, Nov 11) b. Behcet’s syndrome
b. Polyartiritis nodosa c. Herpes Labialis
c. Henoch schonlein purpura d. Fixed drug eruption
d. Kawasaki disease Ref: Harrison 18/e p422 Ref: Neena Khanna 4/e p238, 239; Harrison 18/e p2801
33. Which of following is/are not the feature of Henoch- 41. A 27-year-old male had burning micturation & urethral
schonlein purpura (HSP): (PGI Dec 08) discharge. After 4 weeks he developed joint pains involving
a. Abdominal pain both the knees & ankles, redness of the eyes & skin lesions.
b. Splinter hemorrhage The most probable clinical diagnosis:
c. Thrombocytopenia a. Psoriasis vulgaris (AIIMS May 05, DNB 02)
d. Epistaxis b. Reiter’s syndrome
e. Arthritis Ref: Harrison 18/e p422 c. Behcet’s syndrome
34. All regarding HSP is true except: (PGI 08) d. Sarcoidosis Ref: Neena Khanna 4/e p43
a. Hematuria resolve without treatment 42. A 29-years-old male with a history of long leisure trip
b. Steroids best treat skin lesions presented with right knee pain and swollen joints with
c. Self limiting arthralgia foreign body sensation in eye. The most probable diagnosis
d. Excellent prognosis is:
e. Purpura fulminans Ref: Harrison 18/e p422 a. Sarcoidosis (AI 2009)
35. ‘Pinch purpura’ is diagnostic of: (AIIMS May 05) b. Tuberculosis
a. Systemic 1° amyloidosis c. Reiter’s disease
b. 2° systemic anyloidosis d. Behcet’s disease Ref: Neena Khanna 4/e p43
c. IIP 43. What is not seen in reiters syndrome? (AIIMS Nov 08)
d. Drug induced purpura Ref: Harrison 18/e p418 a. Subcutaneous nodules
36. Treatment of Kawasaki’s disease? (AIIMS May 09, Nov 11) b. Keratoderma blennorrhagicum
a. IV 1g c. Circinate balanitis
b. Steroids d. Oral ulcers Ref: Neena Khanna 4/e p43
c. Thalidomide 44. True regarding reactive arthritis is all except: (PGI 08)
d. Dapsone Ref: Neena Khanna 4/e p282 a. HLA B27 & HIV affects severity
37. A 4-year-old child with high fever developed toxic look, b. Dactylitis & enthesitis
eruptions on trunk & proximal extremities, bilateral bulbar c. Keratoderma mostly on glans
conjunctivitis without discharge and fissuring crusting- red d. Asymmetrical sacroilitis
lips. Due to limb edema walking became difficult. She also e. Onycholysis & hyperkeratosis of nails
had desquamation in perineum & finger tips with cervical Ref: Harrison 18/e p2778-2779
adenopathy. Diagnosis is: 45. All the following are primary cutaneous diseases except:
a. PAN (UP 07, AIIMS 05) a. Psoriasis (AIIMS May 10, AI 09)
b. HSP b. Reiter’s disease

DERMATOLOGY
c. Kawasaki syndrome c. Lichen planus
d. Erythema infectosum d. Icthiosis/Bowen’s disease
Ref: Neena Khanna 4/e p282; Harrison 18/e 2800 Ref: Neena Khanna 4/e p43, 40; Harrison 18/e p2778, 2779
38. Necrotizing lymphadenitis is seen in: (AI 11)
a. Kimura’s disease
b. Hodgkin’s disease
D. PORPHYRIA
c. Castleman’s disease
46. A pinkish red fluorescence of urine with wood’s lamp is
d. Kikuchi disease Ref: Harrison 18/e p1346, 465
seen in: (DP PGMEE 2010)
39. Which of the organisms most commonly causes reactive a. Lead poisoning
arthritis? (AIIMS Nov 08) b. Porphyria cutanea tarda
a. Ureaplasma urealyticum c. Erythromelalgia
b. Group A beta hemolytic streptococci d. Acrocyanosis
c. Borrelia burgdorferi Ref: Neena Khanna 4/e p387
d. Chlamydia Ref: Harrison 18/e p1423

Ans. 32. c. Henoch... 33. c. Thrombocytopenia 34. b. Steroids best... 35. a. Systemic 1°...
36. a. IVI g 37. c. Kawasaki syndrome 38. d. Kikuchi disease 39. d. Chlamydia
40. b. Behcet’s syndrome 41. b. Reiter’s syndrome 42. c. Reiter’s disease 43. a. Subcutaneous...
44. c. Keratoderma... 45. b. Reiter’s disease 46. b. Porphyria cutanea...
152 Jaypee’s Triple A
47. Chandu 32 years male presents with abdominal pain and c. Pyoderma gangrenosum
vomitting. He also complain of some psychiatric symptoms d. Juvenile dermatosis
& visual hallucination. Most likely diagnosis is: Ref: Neena Khanna 4/e p190; Harrison 18/e p423
(AI 01, DNB 04)
a. Hypothyroidism
b. Hyperthyroidism F. PANNICULITIS
c. Hysteria
d. Intermittent porphyria 54. Neonatal fat necrosis (subcutaneous fat necrosis of
Ref: Neena Khanna 4/e p388; Harrison 18/e p3173 newborn) resembles: (AIIMS May 2011)
a. Erythema induratum
48. A girl on sulphonamides developed abdominal pain and
b. Post-steroidal panniculitis
presented to emergency with seizure. What is the probable
c. Lupus panniculitis
cause? (AIIMS Nov 08)
d. Lipodermatosclerosis
a. Acute intermittent porphyria
Ref: Rook’s 8/e p46.1/40; Andrews pediatric dermatology chap-44
b. Congenital erythropoietic porphyria
c. Infectious mononucleosis 55. Erythema nodosum is seen in all of the following except:
d. Kawasaki’s disease Ref: Neena Khanna 4/e p388 a. Pregnancy (AI 11)
b. Tuberculosis
49. A 40 year old farmer with history of recurrent attack of
c. SLE
porphyria complains of itching when exposed to the
d. Chronic pancreatitis
sun and maculopapular rashes on sun exposed area. His
Ref: Neena Khanna 4/e p189
symptoms are exaggerated in summer. The diagnosis is:
a. Seborrheic dermatitis (AIIMS 01) 56. 25-year-old male having fever & malaise since 2 weeks,
b. Contact dermatitis arthritis of ankle joint and tender erytematous nodules over
c. Psoriasis the shin. Diagnosis is: (AIIMS May 2010)
d. Porphyrea cutanea tarda Ref: Neena Khanna 4/e p388 a. Erythema nodosum
b. Hansen’s disease
50. Treatment of choice in the cutaneous complication of
c. Weber-Christian disease
porphyria is: (JIPMER 96, PGI 01)
d. Nodular vasculitis
a. IV dextrose
Ref: Neena Khanna 4/e p188-189
b. IV hematin
c. Beta carotene 57. Which of these statements is false for lesions of erythema
d. Calamine Ref: Neena Khanna 4/e p389 nodosum: (MP 2004)
a. They are considered as hypersensitivity reaction
b. The skin overlying the lesions is red, smooth and shiny
E. NEUTROPHILIC DERMATOSES c. They are usually non tender
d. They can be associated with tuberculosis
51. All are neutrophilic dermatosis except: (PGI June 08) Ref: Neena Khanna 4/e p188-189; Harrison 18/e p2808
a. Subcorneal pustular dermatosis 58. A young female presents with a history of fever and
b. Kimura disease nodular lesion over the shin. Histopathology reveals
c. Granuloma facial foamy histiocytes with neutrophilic infiltration. There is no
d. Sweet’s syndrome evidence of vasculitis. Most probable diagnosis is:
e. Pyoderma gangrenosum a. Sweet’s syndrome (AI 2011)
Ref: Neena Khanna 4/e p190; Harrison 18/e p1346, 1325 b. Erythema nodosum
DERMATOLOGY

52. False about sweet syndrome: (PGI Nov 09) c. Erythema nodosum leprosum
a. May be a/w high fever d. Behcet’s syndrome
b. Neutrophilia not present Ref: Harrison 18/e p2808
c. May be a/w hematological malignancy
d. Pseudovesication
e. Tender erythematous nodule/plaque G. CUTANEOUS DRUG ERUPTIONS
Ref: Neena Khanna 4/e p190
59. Presence of erythema nodosum indicates: (Kerala PG 09)
53. A child with fever had multiple skin lesions, and on
a. Tuberculosis
microscopic examination the skin lesions are seen to have
b. Sarcoidosis
neutrophilic and histiocytic infiltration in the dermis. What
c. Malignancies
is the diagnosis? (AI 09, 11)
d. All of the above
a. Sweet syndrome
Ref: Neena Khanna 4/e p189
b. Behcet’s syndrome

Ans. 47. d. Intermittent... 48. a. Acute intermittent... 49. d. Porphyrea... 50. c. Beta carotene
51. b and c 52. b. Neutrophilia not... 53. a. Sweet syndrome 54. b. Post-steroidal...
55. d. Chronic pancreatitis 56. a. Erythema nodosum 57. c. They are usually... 58. b. Erythema nodosum
59. d. All of the above
Systemic Diseases with Prominent Cutaneous Features... 153
60. A 27-year-old sexually active male develops a vesicobullous 61. Recurrent erythematous plaques on glans penis in a 19
lesion on the glans soon after taking tab paracetamol for yrs old sexually active male which heals with residual
fever. The lesion healed with hyperpigmentation. The most hyperpigmentation, is suggestive of? (AIIMS Nov 09)
likely diagnosis is: (AI 2005) a. Apthous balanitica
a. Behcet’s syndrome b. Fixed drug eruption
b. Herpes genitalis c. Herpes gestations
c. Fixed drug eruption d. Chlamydial infective
d. Pemphigus vulgaris Ref: Neena Khanna 4/e p209
Ref: Neena Khanna 4/e p209; Harrison 18/e p433, 438

DERMATOLOGY

Ans. 60. c. Fixed drug eruption 61. b. Fixed drug eruption


8. LEPROSY (HENSEN’S DISEASE)

A. Leprosy
B. Tuberculosis
C. Other Bacterial Infections
Leprosy (Hensen’s Disease) 155

LEPROSY (HENSEN’S DISEASE) (QUESTIONS)

A. LEPROSY 8. Characteristic feature of borderline leprosy


a. Inverted saucer lesion (AIIMS May 12)
1. An 8-year-old boy from Bihar presents with a 6 months b. ENL
history of an ill defined, hypopigmented slightly atrophic, c. Hypopigmented macule and plaques all over body
leisons face. The most likely diagnosis is: d. Glove and stocking anesthesia
(Delhi 2009 Feb) 9. A 45-year-old male had multiple hypoaesthetic mildly
a. Pityriasis alba erythematous large plaques with elevated margins on trunk
b. Indeterminate leprosy and extremities. His ulnar and lateral popliteal nerves on
c. Morphoea both sides were enlarged. The most probable diagnosis is:
d. Calcium deficiency Ref: Neena Khanna 4/e p256 (AIIMS Nov 03)
2. Satellite lesions are seen in: (COMED 2010) a. Lepromatous leprosy
a. Indeterminate leprosy b. Borderline leprosy
b. Borderline leprosy c. Borderline tuberculoid leprosy
c. Borderline tuberculoid leprosy d. Borderline lepromatous leprosy
d. Lepromatous leprosy Ref: Park PSM 22/e p287-291
Ref: OP Ghai, 7/e p661, table 24.8 and fig 24.23 10. A patient with multiple hypopigmented and hypesthetic
3. Progression to abscess formation in nerves is most patches on lateral aspect of forearm with abundance of
commonly seen in: (Karnataka 2010) AFB and granulomatous inflammation on histology. The
a. Tuberculoid leprosy diagnosis is: (AIIMS May 12)
b. Borderline lepromatous leprosy a. Tuberculoid leprosy
c. Lepromatous leprosy b. Indetermediate leprosy
d. Borderline leprosy c. Borderline leprosy
Ref: Harrison 18/e p1362; Neena Khanna 4/e p259 d. Lepromatous leprosy Ref: Park PSM 22/e p291, 287
4. Which of the following structures is not involved in 11. An 8-year old boy from Bihar presents with a 6 month h/o
leprosy? (J and K 2010) an ill defined hypopigmented slightly atrophic macule on
a. Female genital tract the face. The most likely diagnosis is:
b. Testes a. Ptyriasis alba (AI 05, DNB 01)
c. Eyes b. Indeterminate leprosy
d. Lymph nodes c. Morphea
Ref: Neena Khanna 4/e p257, 258 d. Calcium deficiency Ref: Park PSM 22/e p291
5. Lepromin test is strongly positive in: (MHPGM-CET 2010) 12. 8-year-old boy from Tamil Nadu presents with a white, non
a. LT (Lepromatous leprosy) anaesthetic, non scaly hypopigmented macule on his face,
b. TB (Tuberculoid type) likely diagnosis is: (AI 01, AIIMS 98)
c. BL (Borderline lepromatous) a. Pityriasis alba
d. BB (Borderline tuberculoid) b. Pityriasis versicolour
Ref: Park 22/e p293; Ananthanarayan 9/e p367 c. Indeterminate leprosy

DERMATOLOGY
6. In Schwann cells, mycobacterium leprae divide in how d. Neuritic leprosy Ref: Park PSM 22/e p291
many days? (Maha 2011) 13. In Leprosy which of the following is not seen:
a. 2 – 3 days a. Abnormal EMG (PGI 2K, DNB 03)
b. 11 – 13 days b. Voluntary muscle wasting
c. 22 – 23 days c. Decreased proprioception
d. 32 – 33 days d. Decreased response to tactile sensation
Ref: Hastings Dermatology 2/e p55 e. Increased response to tactile sensation
7. Thin zone of Grentz is seen in: (Kerala 2008) Ref: Harrison 18/e p1361-62
a. Lepromatous leprosy 14. Leprosy affects all organs except: (AIIMS May 10)
b. Sarcoidosis a. Eyes
c. Lymphoma b. Nerves
d. Syphilis c. Uterus
Ref: Roxburgh 18/e p30 d. Ovary Ref: Harrison 18/e p1361,1363

Ans. 1. b. Indeterminate... 2. c. Borderline... 3. a. Tuberculoid... 4. a. Female genital...


5. b. TB... 6. b. 11-13 days 7. a. Lepromatous... 8. a. Inverted saucer lesion
9. d. Borderline... 10. c. Borderline leprosy 11. b. Indeterminate... 12. c. Indeterminate...
13. c. Decreased... 14. c. Uterus
156 Jaypee’s Triple A
15. All are true regarding lepromatous leprosy except: c. Extension of existing lesions
a. Presence of globi (TN 96, CMC 02) d. Development of new skin lesions
b. Subepidermal free zone Ref: Harrison 18/e p1363
c. Decreased cell mediated immunity 24. Type of reaction in Type I Lepra reaction is: (J and K 2011)
d. Presence of granulomas subdermally a. Type I
Ref: Harrison 18/e p1362 b. Type II
16. Lepromin test is used for: (AIIMS Nov 08) c. Type III
a. Diagnosis d. Type IV
b. Treatment Ref: Anathanarayan 9/e 162, 367
c. Prognosis 25. Type-I lepra reaction is commonly seen in:
d. Epidemiological investigation (DP PGMEE 2009)
Ref: Park PSM 22/e p293 a. Histioid leprosy
17. The following test is not used for diagnosis of leprosy: b. Lepromatous leprosy
a. Lepramin test (AIIMS May 06, CMC 09) c. Borderline tuberculoid leprosy
b. Slit skin smear d. Polyneuritic leprosy Ref: Neena Khanna 4/e p262
c. Fine needle aspiration cytology 26. A 27-year-old patient was diagnosed to have borderline
d. Skin biopsy leprosy and started on multibacillary multi-drug therapy.
Ref: Park PSM 22/e p293 Six weeks later, he developed pain in the nerves and
18. Not true about lepromin test is: (AIIMS May 10) redness and swelling of the skin lesions. The management
a. It is diagnostic of his illness should include all of the following, except:
b. Negative in infants < 6 months (AIIMS 04)
c. Used to classify a. Stop anti-leprosy drugs
d. BCG vaccination may convert negative to positive b. Systemic corticosteroids
Ref: Park PSM 22/e p293 c. Rest to the limbs affected
19. Lepromin test is positive in which leprosy: d. Analgesics Ref: Harrison 18/e p7363
a. Lepromatous (AIIMS 2K, DNB 03) 27. The main cytokine, involved in erythema nodusum
b. Indeterminate leprosum (ENL) reaction is:
c. Histoid a. Interleukin – 2 (AIIMS May 2006)
d. Tuberculoid b. Interferon – gamma
Ref: Park PSM 22/e p293 c. Tumor necrosis factor- alpha
20. A 16-year-old student reported for the evaluation of d. Macrophage colony stimulating factor
multiple hypopigmented macules on the trunk and limbs. Ref: Harrison 18/e p1363
All of the following tests are useful in making a diagnosis 28. ENL is seen in which form of leprosy:
of leprosy, except: (AIIMS Nov 03) a. Indeterminate (PGI 05, 04, May 11, AIIMS 92)
a. Sensation testing b. BT
b. Lepromin test c. LL (lepromatous leprosy)
c. Slit smears d. BL
d. Skin biopsy e. TT
Ref: Park PSM 22/e p293 Ref: Harrison 18/e p1362-1363
21. Leprosy commonly affects all organs except: 29. Manifestation of ENL includes all of the following except:
a. Ovary a. Pancreatitis
DERMATOLOGY

b. Skin b. Fever (PGI Dec 05, DNB 08)


c. Nerves c. Hepatitis
d. Eye Ref: Neena Khanna 4/e p256 d. Arthritis
22. Erythema nodosum leprosum is seen in: (Delhi 2009 Feb) e. Cutaneous nodules
a. Lepromatous leprosy Ref: Harrison 18/e p1363
b. Indeterminate leprosy 30. Type-II lepra reaction is found in: (DNB 2005)
c. Borderline tuberculoid leprosy a. BL
d. Tuberculoid leprosy b. LL
Ref: Harrison 18/e p363 c. BT
23. Type I lepra reactions are characterized by: (J and K 2012) d. None of the above
a. Edematous skin lesions Ref: Neena Khanna 4/e p263
b. Satellite lesions

Ans. 15. d. Presence of... 16. c. Prognosis 17. a. Lepramin test 18. a. It is diagnostic
19. d. Tuberculoid 20. b. Lepromin test 21. a. Ovary 22. a. Lepromatous leprosy
23. a. Edematous skin... 24. d. Type IV 25. c. Borderline... 26. a. Stop anti-leprosy...
27. c. Tumor necrosis... 28. c and d 29. a and d 30. b. LL
Leprosy (Hensen’s Disease) 157
31. Drug not used in Type 1 Lepra reaction is:
a. Antipyretics B. TUBERCULOSIS
b. Analgesics
39. Forms of skin tuberculosis include all except:
c. Corticosteroids
a. Lupus vulgaris
d. Thalidomide
b. Scrofuloderma (MHPGM-CET 2008, 2010)
Ref: Neena Khanna 4/e p267, 268
c. Erythema nodosum
32. A Leprosy patient on treatment with MBMDT develops d. Erythema annulare Ref: Harrison 18/e p407
severe Reversal Reaction, 6 months after starting
40. Tuberculosis verrucosa cutis is a form of:
medication. The next step in management is: (J and K 2010)
a. Tuberculid (DNB 07, SGPGI 05)
a. Stop MBMDT for some time
b. Primary tuberculosis
b. Continue MBMDT and add Thalidomide
c. Postprimary tuberculosis with good resistance
c. Continue MBMDT and add systemic steroids
d. Postprimary tuberculosis with poor resistance
d. Continue antileprosy drugs at a reduced dosage without
Ref: Neena Khanna 4/e p253-254
adding any more drugs
Ref: Neena Khanna 4/e p268 41. Most common type of cutaneous T.B. is: (PGI Dec 06)
a. Lupus vulgaris
33. Multibacillary leprosy is treated by: (Kerala PG 09)
b. Scrofuloderma
a. Dapsone alone
c. T.B. verruca cutis
b. Dapsone and rifampicin
d. Erythema induratum
c. Rifampicin + dapsone + clofazimine
Ref: Neena Khanna 4/e p253
d. Rifampicin + clofazimine
Ref: Nenna Khanna 4/e p267 42. Skin manifestations of T.B. (PGI 04)
a. Lupus vulgaris
34. Drug of choice in erythema nodosum leprosum (type II
b. Lupus pernio
lepra reaction):
c. Scrofuloderma
a. Steroid (AI 08, AIIMS 92)
d. Butcher warts Ref: Neena Khanna 4/e p253
b. Thalidomide
c. Clofazimine 43. Tuberculosis of skin is called as: (AIIMS 98, SGPGI 04)
d. Aspirin a. Lupus vulgaris
Ref: Harrison 18/e p1363-1366; Park PSM 22/e p296 b. Lupus pernio
c. Lupus profundus
35. The following drug is not used for the treatment of type II
d. Scrofuloderma Ref: Neena Khanna 4/e p253
lepra reaction: (AIIMS May 06, CMC 08)
a. Chloroquine 44. True about lupus vulgaris: (PGI Dec 04)
b. Thalidomide a. Apple jelly nodule at root of nose
c. Cyclosporine b. TB of skin and mucosa
d. Corticosteroids Ref: Harrison 18/e p1366 c. Also known as scrofuloderma
d. ATT is helpful
36. The most effective drug against M. leprae is: (AI 03)
Ref: Neena Khanna 4/e p221
a. Dapsone
b. Rifampicin 45. A 12-year-old boy had a gradually progressive plaque on
c. Clofazamine a buttock for the last 3 years. The plaque was 15 cm in
d. Prothionamide Ref: Harrison 18/e p1365 diameter, annular in shape, with crusting and induration
at the periphery and scarring at the center. The most likely
37. The first line antileprosy drugs include all except:
diagnosis is:

DERMATOLOGY
a. Dapsone (PGI 97, Delhi 03)
a. Tinea corporis (AIIMS Nov 03)
b. Thiacetazone
b. Granuloma annulare
c. Clofazimine
c. Lupus vulgaris
d. Rifampicin
d. Borderline leprosy
Ref: Harrison 18/e p1365-1366
Ref: Neena Khanna 4/e p221-253
38. Thalidomide is not used in: (AIIMS May and Nov 08)
46. An 8-year-old boy present with well defined annular lesion
a. ENL
over the buttocks with central scarring that is gradually
b. Behcet’s syndrome
progressive over the last 8 months. The diagnosis is:
c. HIV associated oral ulcers
a. Annular psoriasis (AIIMS 01)
d. HIV associated neuropathy
b. Lupus vulgaris
Ref: Harrison 18/e p3460-3461
c. Tinea corporis
d. Chronic granulomatous disease
Ref: Neena Khanna 4/e p253, 221

Ans. 31. d. Thalidomide 32. c. Continue MBMDT... 33. c. Rifampicin +... 34. a. Steroid
35. c. Cyclosporine 36. b. Rifampicin 37. b. Thiacetazone 38. d. HIV associated...
39. d. Erythema annulare 40. c. Postprimary... 41. a. Lupus vulgaris 42. a. Lupus vulgaris
43. a. Lupus vulgaris 44. a, b and d 45. c. Lupus vulgaris 46. b. Lupus vulgaris
158 Jaypee’s Triple A
47. A young boy presented with a lesion over his right buttock
which had peripheral scaling and central clearing with C. OTHER BACTERIAL INFECTIONS
scarring. The investigation of choice would be:
56. Ecthyma gangrenosum is typically due to: (Delhi 2009 Feb)
a. Tzank smear (AIIMS 01)
a. Escherichia coli
b. KOH preparation
b. Pseudomonas aeruginosa
c. Biopsy
c. Staphylococcus aureus
d. Sabouraud agar Ref: Neena Khanna 4/e p252-254
d. Candida albicans Ref: Harrison 18/e p1028
48. Apple-jelly nodules is/are seen in: (PGI Nov 2010)
57. Erysipelas is a skin infection often caused by:
a. Lupus vulgaris
(Karnataka 2010)
b. DLE
a. Erysipelothrix rhusiopathiae
c. Lichen planus
b. Group A beta-hemolytic streptococci
d. Psoriasis Ref: Neena Khanna 4/e p221
c. Trichophyton rubrum
49. A farmer has a single warty lesion on leg. Which of the d. Pseudomonas aeroginosa
following could be most likely lesion: (AIIMS Nov 2010) Ref: Harrison 18/e p1067; Roxburgh 18/e p41
a. Verruca vulgaris
58. Erysipelas is caused by: (J and K 2010)
b. Tuberculosis verrucosa cutis
a. Staph aureus
c. Mycetoma
b. Staph albus
d. Lichen planus hypertrophicus
c. Strep pyogenes
Ref: Neena Khanna 4/e p252
d. E. coli Ref: Neena Khanna 4/e p251
50. Cutaneous (skin) tuberculosis secondary to underlying
59. Normal commensal of skin are: (PGI June 09)
tissue eg lymph node is called as: (AI 99, AIIMS 98, PGI 04)
a. Staphylococcus aureus
a. Lupus vulgaris (DNB 01)
b. Candida
b. Scrofuloderma
c. Propioni bacterium acnes
c. Spina ventosa
d. Diphtheria
d. Tuberculous verrucosa cutis
e. Streptopyogenes
Ref: Neena Khanna 4/e p253
Ref: Neena Khanna 4/e p241
51. Tuberculides are seen in: (AIIMS May 07)
60. 20-year-old male from Jaipur with erythermatous lesion on
a. Lupus vulgaris
cheek with central crusting likely diagnosis is: (AI 01)
b. Scrofuloderma
a. SLE
c. Lichen scrofulososum
b. Lupus vulgaris
d. Erythema nodosum
c. Chillbains
Ref: Rook’s 7/e p28.11, 28.20; Fitzpatrick’s 7/e p1769-74, 272
d. Cutaneous leishmaniasis
52. Which of the following is /are tuberculides: Ref: Neena Khanna 4/e p346
a. Lichen scrofulosorum (PGI June 07, 2K, DNB 03)
61. Involvement of sweat gland, dermal appendages, and hair
b. Lichen nichidus (AIIMS Nov 06)
follicles with epitheloid granuloma are typical features of
c. Lichen aureus
which of the following? (AI 09, DNB 11)
d. Erythema nodosum.
a. Lichen scrofulosum
53. Mycobacterium causing skin ulcer: (PGI 02) b. Miliary TB
a. M. smegmatis c. Papulonecrotic type
b. M. scrofulaceum d. Lupus vulgaris
c. M. ulcerans
DERMATOLOGY

62. False statement about impetigo: (PGI 01)


d. M. fortuitum
a. Mostly caused by staphylococcus or streptococcus or
e. M. marinum Ref: Harrison 18/e p1067
both
54. Skin hazards of swimming are: (PGI 01) b. It predisposes to glomerulonephritis
a. Verrucae c. Produces scar on healing
b. Yoderma gangrenosum d. Erythromycin is drug of choice
c. M. marinum infection e. It is infectious lesion
d. M. ulcerans infection Ref: Neena Khanna 4/e p344 Ref: Roxburgh 18/e p45
55. Tuberculoids are seen in: 63. Impetigo contageosa most commonly due to:
a. Lupus vulgaris a. Group B Streptococcous (Jharkhand 2004)
b. Scrofuloderma b. Staphylococcus
c. Lichen scrofulososum c. Moniliasis
d. Erythema nodosum Ref: Internet d. Streptococcus viridans Ref: Roxburgh 18/e p45

Ans. 47. c. Biopsy 48. a. Lupus vulgaris 49. b. Tuberculosis... 50. b. Scrofuloderma
51. c. Lichen... 52. a. Lichen... 53. c and e 54. a and c
55. c. Lichen... 56. b. Pseudomonas... 57. b. Group A beta... 58. c. Strep pyogenes
59. b and c 60. d. Cutaneous... 61. a. Lichen... 62. c and d
63. b. Staphylococcus
Leprosy (Hensen’s Disease) 159
64. True about impetigo is: (PGI June 08) 67. After 3 days of fever patient developed maculoerythematous
a. Contagious rash lasting for 48 hrs, diagnosis is: (AI 02)
b. Bacterial infection a. Fifth disease
c. Non contagious b. Rubella
d. Honey coloured cast c. Measles
e. Viral infection d. Roseola infantum
Ref: Harrison 18/e p1174-1175; Roxburgh 18/e p45 Ref: Harrison 18/e p1475
65. Desquamation of skin occurs in: (PGI Nov 11) 68. Primary pyodermas are: (PGI May 2011)
a. Erythema infectiosum a. Impetigo contagiosa
b. Kawasaki disease b. Ecthyma
c. Scarlet fever c. Furuncle
d. Toxic shock syndrome d. Pyoderma gangrenosum
e. Infectious mononucleosis e. Impetigo herpetiforms
Ref: Harrison 18/e p2800, 152, 415 Ref: Roxburgh 18/e p45, 46
66. Which of these statements is false for erytherma 69. Which of the following are bacterial infection of skin:
marginatum: (MP 04, DNB 06) a. Pyoderma gangrenosum (PGI June 05, DNB 04)
a. Lesions are serpiginous b. Piedra
b. Characteristically it is an evanescent c. Impetigo contagiosa
c. Rash worsens on application of heat d. Impetigo herpetiformis
d. Rash is itchy e. Ecthyma
Ref: Harrison 18/e p157 Ref: Roxburgh 18/e p45

DERMATOLOGY

Ans. 64. a, b and d 65. b, c and d 66. d. Rash is itchy 67. d. Roseola infantum
68. a, b and c 69. c and e
9. FUNGAL INFECTIONS, SCABIES
AND PEDICULOSIS

A. Fungal Infections
B. Scabies
C. Pediculosis
D. Miscellaneous
Fungal Infections, Scabies and Pediculosis 161

FUNGAL INFECTIONS, SCABIES AND PEDICULOSIS (QUESTIONS)

A. FUNGAL INFECTION 8. Grisefulvin given for the treatment of fungal infection in


the figure nail dermatophytosis for how much duration:
1. Dermatophytes are: (PGI 06, 03) a. 4 weeks (AI 02, CMC 05)
a. Sporothrix b. 6 weeks
b. Tinea versicularis c. 2 months
c. Microsporidium d. 3 months
d. Trichophyton rubrum Ref: Neena Khanna 4/e p290; Roxburgh 18/e p43
e. All of the above Ref: Neena Khanna 4/e p283 9. A 22-years-old male patient presents with a complaints
2. Dermatophytes infection to: (Bihar 06, DNB 09) of severe itching and white scaly lesions in the groin for
a. Superficial past month. Which of the following is most likely to be the
b. Subdermal causative agent: (AIIMS Nov 03, JIPMER 05)
c. Subfascial a. Trichophyton rubrum
d. Muscular Ref: Neena Khanna 4/e p283 b. Candida albicans
3. Most common organisim causing T. capitis is c. Candida glabrata
a. Trichophyton tonsurans (AI 01, DNB 03) d. Malassezia furfur
b. Microsporum Ref: Neena Khanna 4/e p283-286; Roxburgh 18/e p40, 41, 42
c. Epidermophyton 10. A 36-years-old factory worker developed itchy annular
d. Candida albicans scaly plaques in both groins. Application of a corticosteroid
Ref: Neena Khanna 4/e p283 ointment led to temporary relief but the plaques continued
4. A 10-yr-old boy presented with painful boggy swelling to extend at the periphery. The most likely diagnosis is:
of scalp, multiple sinuses with purulent discharge, easily (AI 05, CMC 07)
pluckable hair, and lymph nodes enlarged in occipital a. Erythema annulare centrifugam
region. Which one of the following would be most helpful b. Granuloma annulare
for diagnostic evaluation? (AIIMS Nov 09, AI 2001) c. Annular lichen planus
a. Biopsy and Giemsa staining d. Tenia cruris Ref: Neena Khanna 4/e p285
b. Bacterial culture 11. Tinea incognito is seen with: (PGI 99, Calcutta 02)
c. KOH mount a. Steroid treatment
d. Patch test, Gram staining and Tzank smear b. 1% BHi3
Ref: Neena Khanna 4/e p288 c. 5% permethrin
5. An eleven year old boy is having tinea capitis on his scalp. d. Antibiotics
The most appropriate line of treatment is: (AI 2003) Ref: Neena Khanna 4/e p285; Roxburgh 18/e p43
a. Oral griseofulvin therapy 12. A 30-year-old female presents with history of itching under
b. Topical griseofulvin therapy right breast. On examination annular ring lesion was
c. Shaving of the scalp present under the breast. The diagnosis is: (AIIMS May 02)
d. Selenium sulphide shampoo a. Trichophyton rubrum
Ref: Neena Khanna 4/e p290; Roxburgh 18/e p43 b. Candida albicans

DERMATOLOGY
6. Treatment of tinea unguium: (PGI June 05) c. Epidermophyton
a. Fluticasone d. Microsporum Ref: Neena Khanna 4/e p283-286
b. Itraconazole 13. The test likely to help in diagnosis of a patient who presents
c. Oleamine oil with an itchy annular plaque on the face is:
d. Turbinafin a. Gram’s stain (AI 2003)
e. Neomycin b. Potassium hydroxide mount
Ref: Neena Khanna 4/e p280 c. Tissue smear
7. DOC for tenia ungum: (AI 93, UP 05, DNB 02) d. Wood’s lamp examination Ref: Neena Khanna 4/e p288
a. Ampthotericin B 14. Which of the following drugs is not antifungal? (AI 08)
b. Miconazole a. Capofungin
c. Greseofulvin b. Undecylenic acid
d. Nystatin c. Ciclopirox
Ref: Neena Khanna 4/e p290 d. Clofazimine Ref: Neena Khanna 4/e p289, 290

Ans. 1. c and d 2. a. Superficial 3. b. Microsporum 4. c. KOH mount


5. a. Oral griseofulvin... 6. b. Itraconazole 7. c. Greseofulvin 8. d. 3 months
9. a. Trichophyton... 10. d. Tenia cruris 11. a. Steroid treatment 12. a. Trichophyton
13. b. Potassium... 14. d. Clofazimine
162 Jaypee’s Triple A
15. An otherwise healthy male presents with a creamy curd like 24. Scabies in children differs from that in adults in that it
white patch on the tongue. The probable diagnosis is: affects: (JIPMER 2K, DNB 09)
a. Candidiasis (AI 2010) a. Webspace
b. Histoplasmosis b. Face
c. Lichen planus c. Genitalia
d. Aspergillosis Ref: Neena Khanna 4/e p294 d. Axilla Ref: Neena Khanna 18/e p343
16. Commonest fungal infection of the female genitalia in 25. Adult scabies is characterized by:
diabetes is: (Karnataka 98, DNB 2K) a. Involve palm and soles (PGI Dec 04)
a. Cryptococcal b. Involve face
b. Madura mycosis c. Involve anterior abdomen
c. Candidial d. Involve web space
d. Aspergellosis Ref: Neena Khanna 4/e p293 e. Involve genitalia Ref: Roxburgh 18/e p62
26. A 9-month-old child has multiple itchy papulovesicular
lesions on face, trunk, palm and sole. Similar lesions are
B. SCABIES also seen in the younger brother. Which of the following is
most possible diagnosis?
17. Oral medication used in the treatment of scabies:
a. Papular urticaria (AIIMS Nov 02, 95, 96)
a. Albendazole (Karnataka 2011)
b. Scabies
b. Metronidazole
c. Atopic dermatitis
c. Ketoconazole
d. Allergic contact dermatitis
d. Ivermectin Ref: KD Tripathi 7/e p904
Ref: Neena Khanna 4/e p341-344
18. Scabies was rediscovered by: (Kerala 2008)
27. An infant presented with itchy eczematous crusted lesions
a. Salani
with exudation on palm, sole, glans penis and face. All are
b. Nikolsky
true except: (PGI 02)
c. Renucci
a. Family should be examined
d. Max delbruk Ref: Internet
b. Only patient needs drug treatment
19. Scabies, an infection of the skin caused by Sarcoptes c. All clothing and linen should be discarded or burnt
scabiei, is an example of: d. Distribution of lesion help in diagnosis making.
a. Water borne disease (AIIMS Nov 02) e. Drug should be applied to affected areas for whole day
b. Water washed disease Ref: Neena Khanna 4/e p341-344
c. Water based disease
28. An infant presenting with itchy lesions over groin and
d. Water related disease Ref: K Park 22/e p659
prepuce all is indicated except: (AI 01)
20. Incubation period of scabies is: (Calcutta 2K, DNB 04) a. Bathe and apply scabicidal solution
a. 7 days b. Treatment of all family members
b. 2 weeks c. Dispose all clothes by burning
c. 4 weeks d. IV antibiotics Ref: Neena Khanna 4/e p344
d. 2-3 days Ref: Roxburgh 18/e p60
29. An 8-month-old child presented with itchy, exudative lesions
21. Characteristic lesion of scabies is: (Kerala 87, Bihar 03) on the face, palms and soles. The siblings also have similar
a. Burrow (UPSC 04) complaints. The treatment of choice is such a patient is:
b. Fissure a. Systemic ampicillin (AI 03)
c. Vesicle b. Topical betamethasone
DERMATOLOGY

d. Papule Ref: Roxburgh 18/e p61 c. Systemic prednisolone


22. Most severe form of scabies: d. Topical permethrin Ref: Neena Khanna 4/e p344
a. Norwegian scabies (AP 98, UPSC 96, DNB 01) 30. Ivermectin in indicated in the treatment of:
b. Neular scabies a. Syphilis (AIIMS May 2006)
c. Animal scabies b. Scabies
d. Genital scabies c. Tuberculosis
e. Generalised scabies Ref: Roxburgh 18/e p62 d. Dermatophytosis Ref: Neena Khanna 4/e p344
23. Circle of hebra is associated with: (PGI June 08) 31. Drugs/treatment used in scabies are: (PGI Nov 11)
a. Syphilis a. Crotamiton
b. Scabies b. Permethrin
c. Leprosy c. Lindane
d. Lichen planus d. Gammexene
e. Washing of body clothing Ref: Neena Khanna 4/e p344

Ans. 15. a. Candidiasis 16. c. Candidial 17. d. Ivermectin 18. c. Renucci


19. d. Water related... 20. c. 4 weeks 21. a. Burrow 22. a. Norwegian scabies
23. b. Scabies 24. b. Face 25. a, d and e 26. b. Scabies
27. a, c and e 28. d. IV antibiotics 29. d. Topical permethrin 30. b. Scabies
31. All of the above
Fungal Infections, Scabies and Pediculosis 163
32. A 6-month-old infant presented with multiple erythematous 40. Donovan bodies with granuloma is seen in: (Raj 2009)
papules and exudative lesions on the face, scalp, trunk and a. Chlamydia
few vesicles on palms and soles for 2 weeks. His mother has b. H.ducreyi
H/o itchy lesions. The most likely diagnosis is: c. H.influenza
a. Scabies (AIIMS May 12, May 05, AI 06) d. Leishmaniasis Ref: Roxburgh 18/e p51
b. Infantile eczema (atopic dermatitis) 41. Tinea capitis caused by: (AP 2011)
c. Infantile seborrheic dermatitis a. Microsporum species
d. Impetigo contagiosa b. Epidermophton
e. Seborrheic dermatitis Ref: Neena Khanna 4/e p341-344 c. Tricophyton
d. All of the above Ref: Roxburgh 18/e p41
C. PEDICULOSIS 42. Which of the following are fungal infection of skin:
a. Sporotrichosis (PGI June 05)
33. Vagabond’s disease is: (Calcutta 2K, DNB 01) b. Molluscum contagiousm
a. Pediculosis corposis c. Madura foot
b. Scabies d. Tinea Ref: Roxburgh 18/e p44, 40
c. Eczema 43. Which of the following stains is used to study fungal
d. Ringworm Ref: Neena Khanna 4/e p339 morphology in tissue sections: (AI 10)
a. PAS
b. Von- kossa
D. MISCELLANEOUS c. Alizarin Red
d. Masson’s Trichrome Ref: Ananthanarayan 9/e p592
34. Jock itch is caused by: (Karnataka 2010) 44. Wavelength of light produced by wood’s Lamp is:
a. Tinea pedis a. 320 nm (PGI 99, AIIMS 01)
b. Tinea capitis b. 360 nm
c. Tinea unguium c. 400 nm
d. Tinea cruris Ref: Davidson’s, 20/e p1297 d. 480 nm
35. Infectious cause of erythematous rashes is: e. 760-800 nm Ref: Neena Khanna 4/e p289, 14
a. Measles (Karnataka 2010) 45. The wood’s lamp filter is made of: (AIIMS 99, PGI 2K)
b. Typhoid fever a. Tin and chromium oxide
c. Yellow fever b. Nickel oxide and silica
d. Malaria Ref: Harrison 18/e p1600-1603 c. Copper oxide and Barium oxide
36. Potassium iodide is useful in the treatment of: d. Zinc oxide
a. Sporotrichosis (DP PGMEE 2009) Ref: Internet
b. Impetigo 46. A pinkish red flurorescence of urine with wood’s lamp is
c. Viral warts seen in: (JIPMER 99, PGI 2K)
d. Dermatitis herpetiformis a. Lead poisoning
Ref: Ananthnarayan 9/e p603; Neena Khanna 4/e p297, 298 b. Porphyria cutanea tarda
37. Griseofulvin is not useful in one of the following: c. Erythromelagia
a. Tinea capitis (DP PGMEE 2010) d. Acrocyanosis
b. Tinea cruris Ref: Neena Khanna 4/e p388

DERMATOLOGY
c. Tinea versicolor 47. Wood’s lamp light is used in the diagnosis of:
d. Tinea pedis Ref: Roxburgh 18/e p40-44 a. Tinea capitis (AIIMS May 02)
38. ‘Coral red’ fluorescence on Wood’ lamp is seen in: b. Candida albicans
a. Erythrasma (MHPGM-CET 2008, 2010) c. Histoplasma
b. Erysipelas d. Cryptococcus
c. Pityriasis versicolor Ref: Neena Khanna 4/e p289
d. Tinea corporis Ref: Neena Khanna 4/e p243 48. Definitive diagnosis of sporotrichosis generally depends
39. Pautrier’s micro abscess in caused by: (Raj 2008, 2009) on:
a. Mycosis fungoides a. Serology
b. Psoriasis b. Culture
c. Lichen planus c. Biopsy
d. Tuberous sclerosis Ref: Neena Khanna 4/e p378 d. KOH preparation from the lesion
Ref: Neena Khanna 4/e p297

Ans. 32. a. Scabies 33. a. Pediculosis... 34. d. Tinea cruris 35. a. Measles
36. a. Sporotrichosis 37. c. Tinea versicolor 38. a. Erythrasma 39. a. Mycosis fungoides
40. d. Leishmaniasis 41. a. Microsporum... 42. a, c and d 43. a. PAS
44. b. 360 nm 45. b. Nickel oxide... 46. b. Porphyria... 47. a. Tinea capitis
48. b. Culture
164 Jaypee’s Triple A
49. Erythrasma is a superficial infection caused by: c. Mixed myocotic infections
a. Corynebacterium diphtheriae d. Candidiasis only
b. Corynebacterium minutissiumum Ref: Davidson 21/e p1265
c. Pseudomonas 51. All are used in the treatment of leishmaniasis except:
d. Bacillus anthracis a. Hydoroxychloroquine
Ref: Neena Khanna 4/e p242 b. Miltefosine
50. Selenium sulfide is indicated for treating: c. Paromomycin
a. Tinea versicolor d. Rifabutin
b. Tinea corporis Ref: Neena Khanna 4/e p348
DERMATOLOGY

Ans. 49. b. Corynebacterium... 50. a. Tinea versicolor 51. d. Rifabutin


10. VENEREAL
(SEXUALLY TRANSMITTED)
DISEASES

A. Warts
B. Lymphogranuloma Venereum
C. Donovanosis
D. Hemophilus Ducreyi
E. Gonorrhea
F. Syphilis
G. Miscellaneous
166 Jaypee’s Triple A

VENEREAL (SEXUALLY TRANSMITTED DISEASES) (QUESTIONS)

A. WARTS 9. Treatment of choice for genital warts in pregnancy?


a. Salicylic acid with lactic acid solution
1. Causative organism of molluscum contagiosum is: b. Podophyllin (AIIMS Nov 09)
a. Papova virus (DP PGMEE 2010) c. Imiqimod
b. Pox virus d. Cryotherapy Ref: Neena Khanna 4/e p274
c. Orthomyxo virus 10. HPV vaccine is: (AIIMS Nov 09)
d. Parvovirus a. Monovalent
Ref: Roxburgh 18/e p55; Neena Khanna4/e p274 b. Bivalent
2. Imiquimod used in treatment of anogenital warts acts c. Quadrivalent
mainly as: (MHPGM-CET 2010) d. Bivalent and quadrivalent Ref: Harrison 18/e p1036
a. Antifungal
b. Antiviral
c. Immunostimulator B. LYMPHOGRANULOMA VENEREUM
d. Keratolytic Ref: Harrison 18/e p1483 and Internet
11. Groove Sign of Greenblatt is found in: (WB PG 08)
3. Condyloma acuminata is caused by: (Maha 2011) a. LGV
a. HPV b. Chancroid
b. HSV c. Donovanosis
c. HIV d. Elephantiasis Ref: Neena Khanna 4/e p310
d. Parvo virus Ref: Harrison 18/e p1065
12. LGV is caused by: (AIIMS 99, PGI 98, Kerala 99, Bihar 05)
4. Verrucosa vulgaris is caused by: a. Chalamydia trachomatis (DNB 06)
a. HPV (Bihar 04, DNB 05) b. Hemophylus ducrei
b. EBV c. HTLV type II
c. CMV d. Donovanosis granulomatis Ref: Neena Khanna 4/e p310
d. HIV Ref: Neena Khanna 4/e p369
13. Genital elephantiasis is caused by: (AI 02)
5. Genital warts (condyloma accuminata) are most commonly a. Donovanosis
caused by which of the following serotypes of HPV? b. Congenital syphilis
a. HPV 6 (AIIMS May 08, 04, 03) c. Herpes genitalis
b. HPV 16 d. Lymphogranuloma venerum
c. HPV 18 Ref: Neena Khanna 4/e p310, 311
d. HPV33 Ref: Neena Khanna 4/e p270
14. Frei’s test is diagnostic of:
6. Immunomodulator used in treatment of genital warts is: a. Lymphogranuloma venerum
a. ATRA (AI 08) b. Lymphogranuloma inguinale
b. Podophyllin c. Donovanosis
c. Imiquimod d. Soft chancre Ref: Neena Khanna 4/e p310, 311
d. Prednisolone Ref: Neena Khanna 4/e p274
7. Podophylline is used in treatment of
C. DONOVANOSIS
DERMATOLOGY

a. Plantar warts (AIIMS 92, 94, 02, 04)


b. Palmar warts
c. Comdylomata accuminata (genital wart) 15. Causative organism for donovanosis is: (DP PGMEE 2009)
d. Condylomata lata Ref: Neena Khanna 4/e p274 a. E. coli
8. Regarding podophyllin resin which of following statement b. Chlamydia trachomatis
is true: (PGI Dec 08) c. H. ducreyi
a. Derived from plant source d. Calymmatobacterium granulomatis
b. Safe in pregnancy Ref: Neena Khanna 4/e p309
c. Teratogenic 16. Tenderness is uncommon in: (MHPGM-CET 2010)
d. High recurrence rate of wart after podophyllin resin treatment a. Herpes genitalis
e. Individual variation in response rate b. Chancroid
Ref: Harrison 18/e p1481 c. Donovanosis
d. All of the above Ref: Neena Khanna 4/e p309

Ans. 1. b. Pox virus 2. c. Immunostimulator 3. a. HPV 4. a. HPV


5. a. HPV 6. c. Imiquimod 7. c. Comdylomata... 8. a, c, d and e
9. d. Cryotherapy 10. d. Bivalent and... 11. a. LGV 12. a. Chalamydia...
13. d. Lymphogranuloma... 14. a. Lymphogranuloma... 15. d. Calymmatobacterium... 16. c. Donovanosis
Venereal (Sexually Transmitted) Diseases 167
17. Ucler on penis which bleeds on touch, is feature that best 24. Painful lymphadenopathy is seen in: (PGI 02)
fits in which of the following? (Maha 2011) a. Donovanosis
a. Donovanosis b. Syphilis
b. Syphilis c. Chancroid
c. Herpes genitalis infection d. Herpes simplex
d. Lymphogranuloma venereum e. Gonorrhea Ref: Neena Khanna 4/e p307, 278
Ref: Neena Khanna 4/e p309 25. A 30-year-old male presented with ulcerative lesion on
18. About Donovanosis false is: (AP 2010) glans penis. Wright- Giemsa stain showed 1-2 rounded
a. Caused by Leishmania donovani structure in macrophage vacuoles. What is the etiology:
b. Presents as bleeding beefy red granulomatous ulcer a. Chlamydia trachomatis (AIIMS May 10)
c. Presents with lymphoedema of tissue during active stage b. N. gonorrhoea
d. Azithromycin is the first choice of therapy c. H. ducreyi
Ref: Neena Khanna 4/e p309 d. Calymmatobacterium granulomatis
19. A 30-year-old male patient has a large, spreading and Ref: Neena Khanna 4/e p309
exuberant ulcer with bright red granulation tissue over the 26. 19-years-male develops painless penile ulcers 9 days after
glans penis. There was no lymphadenopathy. The most sexual intercourse with a professional sex worker likely
likely causative organism is: (AIIMS Nov 03) diagnosis is:
a. Treponema pallidum a. Chancroid (AI 01)
b. Herpes simplex virus type 1 b. Herpes
c. Herpes simplex virus type 2 c. Chancre
d. Calymmatobacterium granulomatis d. Traumatic ulcer Ref: Neena Khanna 4/e p307
Ref: Neena Khanna 4/e p309 27. A 23-year-old male had unprotected sexual intercourse with
a commercial sex worker. Two weeks later, he developed
a painless, indurated ulcer on the glans which exuded
D. HEMOPHILUS DUCREYI clear serum on pressure. Inguinal lymph nodes in both
groins were enlarged and not tender. The most appropriate
20. A 20-years-old male developed multiple tender non-
diagnostic test is: (AIIMS 04)
indurated bleeding ulcers over prepuce 4 days after sexual
a. Gram’s stain of ulcer discharge
exposure. The most probable diagnosis in this patients is:
b. Darkfield microscopy of ulcer discharge
a. Primary syphilis (Delhi 2009 Feb)
c. Giemsa stain of lymph node aspirate
b. Lymphogranuloma venereum
d. ELISA for HIV infection Ref: Neena Khanna 4/e p307-311
c. Chancroid
d. Donovanosis 28. A 24-year-old male presents to a STD clinic with a single
Ref: Neena Khanna 4/e p307 painless ulcer on external genitalia. The choice of laboratory
test to look for the etiological agent would be:
21. The following is NOT true of chancroid: (AP 2012)
a. Scrappings from ulcer for culture on chocolate agar with
a. The bubo is multilocular abscess
antibiotic supplement (AIIMS May 03)
b. The overlying skin is erythematous and shiny
b. Serology for detection of specific IgM antibodies
c. If untreated, the bubo ruptures to form a single sinus
c. Scrappings from ulcer for dark field microscopy
d. The opening of the sinus may enlarge to form a giant
d. Scrappings from ulcer for tissue culture
ulcer
Ref: Neena Khanna 4/e p309-311
Ref: Neena Khanna 4/e p307-309

DERMATOLOGY
29. H. ducreyi causes:
22. Reliable test for chancroid detection: (AP 98, AI 98)
a. Chancroid
a. Skin test (DNB 02)
b. Hard chancre
b. Biopsy
c. Pustule
c. Gram stained smear
d. Painless chancre Ref: Neena Khanna 4/e p307
d. Clinical examination
Ref: Neena Khanna 4/e p308
23. A man having multiple, painful, indurated, undermined, E. GONORRHEA
sloughed edged glans which occurred 5 days after
exposures; most likely diagnosis is: 30. The term gonorrhea was coined by: (Kerala 2008)
a. Chancroid (AI 08) a. Ducrei
b. Primary chancre b. Turner
c. Herpes genitalis c. Galen
d. LGV Ref: Neena Khanna 4/e p306, 307 d. Neisser Ref: Ananthanarayan, 7/e p226

Ans. 17. a. Donovanosis 18. a. Caused by... 19. d. Calymmatobacterium... 20. c. Chancroid
21. a. The bubo is... 22. c. Gram stained smear 23. a. Chancroid 24. c and d
25. d. Calymmato... 26. c. Chancre 27. b. Darkfield microscopy... 28. c. Scrappings from...
29. a. Chancroid 30. c. Galen
168 Jaypee’s Triple A
31. The main feature of gonorrhea is: (Karnataka 98, DNB 01) 40. A 40-year-old female presented with numerous, nonitchy,
a. Purulent discharge per urethra erythematous scaly papules (lesions) on trunk, with few
b. Inguinal adenitis oral white mucosal plaques. She also had erosive lesions in
c. Ulcer over glans penis perianal area. The probable diagnosis is: (AI 12)
d. Rashes Ref: Neena Khanna 4/e p315, 317 a. Psoriasis
32. The syndromic management of urethral discharge includes b. Secondary syphilis
treatment of: (AI 03) c. Lichen planus
a. Neisseria gonorrheae and herpes genitalis d. Disseminated candidiasis Ref: Neena Khanna 4/e p306
b. Chlamydia trachomatis and herpes ganitalis 41. Secondary syphilis manifested by: (PGI 03)
c. Neisseria gonorrheae and Chlamydia trachomatis a. Painless lymphadenopathy
d. Syphilis and chancroid b. Pruritic rash
Ref: Neena Khanna 4/e p315-317 c. Mucosal erosion
d. Asymptomatic rash
e. Mostly asymptomatic Ref: Neena Khanna 4/e p306
F. SYPHILIS 42. A 23-year-old college student has asymptomatic and
hyperpigmented macules on both palms for three weeks.
33. Thymus gland abscess in congenital syphilis is called:
The most appropriate diagnostic test is: (AIIMS 04)
a. Fouchier’s abscess (J and K 2010)
a. Veneral Diseases Research Laboratory (VDRL) test
b. Politzeri abscess
b. Skin biopsy
c. Douglas abscess
c. Serum cortisol levels
d. Dubosis abscess Ref: Internet
d. Assay for arsenic in skin, hair and nails
34. Which of the following skin lesion is not seen in secondary Ref: Neena Khanna 4/e p305, 306
syphilis? (J and K 2010)
43. Treponema pallidum isolation from CSF is maximum in
a. Macule
which stage of syphilis? (AIIMS May 09)
b. Papule
a. Primary syphilis
c. Bullae
b. Secondary syphilis
d. Pustule Ref: Neena Khanna 4/e p302
c. Tertiary syphilis
35. The following is the most common ulcerative STD in India: d. Tabes dorsalis
a. Syphilis (J and K 2011) Ref: Neena Khanna 4/e p306, 302; Harrison 18/e p1382
b. Chancroid
44. A patient has syphilis since 2 years. CSF examination was
c. Herpes genitalis
done and treatment started. Which of the following test is
d. Granuloma inguinale Ref: Neena Khanna 4/e p301
most useful in monitoring treatment: (AIIMS Nov 09)
36. Which stage of syphilis is most contagious? a. TPI
a. Primary (DP PGMEE 2009) b. VDRL
b. Secondary c. FTA
c. Early latent d. Dark ground microscopy Ref: Neena Khanna 4/e p305, 306
d. Later latent Ref: Neena Khanna 4/e p302
45. Most specific test for syphilis: (AIIMS May 10)
37. Ollendorf’s sign is seen in: a. VDRL
a. Primary syphilis (SS CET 2009, MHPGM-CET 2010) b. RPR
b. Secondary syphilis c. FTA-ABS
c. Latent syphilis d. Kahn’s test Ref: Neena Khanna 4/e p305
DERMATOLOGY

d. Neurosyphilis
46. Jarisch Herxheimer reaction is commonly seen in:
38. ‘Chancre redux’ is a clinical feature of: a. Early syphilis (DNB 03, SGPGI 05, PGI 98)
a. Early relapsing syphilis b. Late congenital syphilis
b. Late syphilis (AIIMS May 06) c. Latent syphilis
c. Chancroid d. Syphilis of cardiovascular system Ref: Harrisons 18/e p1388
d. Recurrent herpes simplex infection
47. A young man presents to the emergency department with
39. A boy with multiple bullous lesions over trunk a maculopapular rash 2 weeks after healing of a painless
and periostitis on X-rays. What should be the next genital ulcer. The most likely etiological agent is: (AI 11)
investigation: (DNB 10, AIIMS Nov 11) a. Treponema pallidum
a. VDRL of mother and child b. Treponema pertunae
b. PCR for maternal TB c. Chalmydia trachomatis
c. HBsAg screening d. Calymmatobacter granulomatis
d. ELISA of mother and Child Ref: Neena Khanna 4/e p306 Ref: Neena Khanna 4/e p300

Ans. 31. a. Purulent discharge... 32. c. Neisseria... 33. d. Dubios abscess 34. c. Bullae
35. a. Syphilis 36. b. Secondary 37. b. Secondary syphilis 38. a. Early relapsing...
39. a. VDRL of mother... 40. b. Secondary... 41. a, c and d 42. a. Veneral Diseases...
43. b. Secondary syphilis 44. b. VDRL 45. c. FTA-ABS 46. a. Early syphilis
47. a. Treponema pallidum
Venereal (Sexually Transmitted) Diseases 169
48. Drug of choice for syphilis in a pregnant women: (AI 12) a. Lymphoma
a. Erythromycin b. Tuberculosis
b. Penicillin c. Non specific
c. Tetracycline d. Viral Ref: Harrisons 18/e p1519-1525
d. Ceftriaxone Ref: Neena Khanna 4/e p307 52. Recurrent balanoposthitis seen in: (PGI 02)
49. All is true about syphilis except: (DNB 11, PGI May 12) a. DM
a. Seropositive infant not treated at birth if mother received b. Herpes simplex
penicillin in 3rd trimester c. Smoking
b. For neurosyphilis FTA-ABS is sensitive; VDRL diagnostic d. Alcohol
but CSF pleocylosis is best treatment response guide. e. Bad hygiene
c. HIV patients are less likely to become VDRL nonreactive Ref: Dermatological signs of internal diseases by Collen, Jorizzo p177;
after treatment IADVL- Textbook and Atlas of Dermatology 2/e Vol I p1492; Fitzpatrick’s
d. EIA+, RPR+, indicate past or current infection 6/e p2164-2212; Rooks 7/e p30.1-30.30, 25.20-39
e. Sulfonamides and quinolones are 2nd line drugs 53. Genital ulcer is/are caused by: (PGI Nov 2009)
Ref: Neena Khanna 4/e p300-307 a. Human papilloma virus
b. Herpes simplex virus
c. HIV
G. MISCELLANEOUS d. Treponema pallidum
e. Lymphogranuloma venereum
50. In HIV infection all of the following conditions constitute
Ref: Neena Khanna 4/e p300, 310, 278
AIDS-Defining disease except: (J and K 2012)
a. Esophageal candidiasis 54. Syndromic management of genital ulcer syndrome in India
b. Oral hairy leukoplakia includes: (AIIMS Nov 11)
c. CMV retinitis a. Chancroid and primary chancre
d. Pulmonary TB Ref: Harrisons 18/e p1543-1552 b. Chancroid and herpes simplex
c. Chancroid, primary chancre and herpes simplex
51. The most common cause of lymph node enlargement in
d. Herpes simplex and primary chancre
AIDS is: (J and K 2010)
Ref: Neena Khanna 4/e p322, 300

DERMATOLOGY

Ans. 48. a. Erythromhycin 49. a. Seropositive... 50. b. Oral hairy... 51. d. Viral
52. a. DM 53. b, d and e 54. c. Chancroid, primary...
11. GENODERMATOSES

A. Xeroderma Pigmentosum
B. Incontinentia Pigmenti
C. Neurofibromatosis
D. Ichthyosis
E. Miscellaneous
Genodermatoses 171

GENODERMATOSES (QUESTIONS)

A. XERODERMA PIGMENTOSUM 7. The mode of inheritance of incontinentia pigmenti is:


a. Autosomal dominant (Kerala 04, J&K 07, UP 06)
1. Defective DNA repair is present in: (AIIMS Nov 09) b. Autosomal recessive
a. Albinism c. X- linked dominant
b. Xeroderma pigmentosum d. X- linked recessive Ref: Neena Khanna 4/e p36
c. Vitiligo 8. True about incontinentia pigmenti include the following
d. Icthyosis Ref: Neena Khanna 4/e p36 except: (AI 09, CMC 08, AIIMS May 11)
2. Genodermal disease that can cause skin malignancy are: a. X-linked dominant
a. Xeroderma pigmentosum (DNB 98, PGI 03) b. Primary skin abnormality
b. Neurofibromatosis c. Avascularity of peripheral retina
c. Actinic keratosis d. Ocular involvement is seen in almost 100% cases and is
d. Porphyria cutanea tarda Ref: Neena Khanna 4/e p37 typically unilateral
3. Cells cultured from patients with this disorder exhibit Ref: Neena Khanna 4/e p36; Harrison 18/e p413
low activity for the nucleotide excision repair process.
This autosomal recessive genetic disease includes marked
C. NEUROFIBROMATOSIS
sensitivity to sunlight (Ultra voilet light) with subsequent
formation of multiple skin cancers and premature death, 9. Plexiform neurofibromatosis commonly affects the
the disorder is: …………… cranial nerve: (DP PGMEE 2010)
a. Acute intermittent porphyria a. 7th
b. Alkaptonuria (Karnataka 98, TN 03, UP 02) b. 5th
c. Xeroderma pigmentosa c. 6th
d. Ataxia – telangiectasa d. 8th Ref: Internet
Ref: Neena Khanna 4/e p36, 37; Harrison 18/e p488, 496;
10. Neurofibromatosis all are true except: (AIIMS May 09)
Roxburgh 18/e p230
a. Autosomal recessive
4. About Xeroderma pigmentosa, true statement is: b. Scoliosis
a. Is a non hereditary disorder c. Neurofibroma
b. Photosensitivity is absent d. Association with cataract
c. DNA repair is defective
11. A patient had seven irregular hyperpigmented macules on
d. Tumor survelliance is needed
the trunk and multiple small hyperpigmented macules in
Ref: Neena Khanna 4/e p36
the axillae and groins since early childhood. There were no
other skin lesions. Which is the most likely investigation to
B. INCONTINENTIA PIGMENTI support the diagnosis?
a. Slit lamp examination of eye (AI 06, DNB 02)
5. A girl child with verrucous lesions at an age of 2 weeks later b. Measurement of intraocular tension
on developed linear bands of hyperkeratotic papules and c. Examination of fundus
nodules followed by whorled pigmentation. Her mother d. Retinal artery angiography

DERMATOLOGY
had history of in utero child death and hypopigmented Ref: Neena Khanna 4/e p34
atrophic linear lesions. The diagnosis is: 12. Child with h/o hypopigmented macule on back, infantile
a. Neurofibromatosis (AIIMS May 08) spasm and delayed milestone has:
b. Xeroderma pirmentosa a. NF (AIIMS 02, CMC 05, DNB 07)
c. Tuberous sclerosis b. Sturge weber syndrome
d. Incontinentia pigmenti c. Tuberous sclerosis
Ref: Neena Khanna 4/e p36 d. Nevus anemicus Ref: Neena Khanna 4/e p33
6. 2-month-old girl present with verrucous plaque on the 13. All are seen in tuberous sclerosis except: (AI 2K)
trunk. What is your most probable diagnosis? a. Iris nodule
a. Incontinentia pigmenti (AIIMS Nov 08, DNB 01) b. Renal cortical cyst
b. Darier disease c. Rhabdomyoma of heart and lung
c. Congenital naevus d. Adenoma sebaceum
d. Icthyosis Ref: Neena Khanna 4/e p36 Ref: Neena Khanna 4/e p33, 34

Ans. 1. b. Xeroderma... 2. a. Xeroderma... 3. c. Xeroderma... 4. c. DNA repair is...


5. d. Incontinentia... 6. a. Incontinentia... 7. c. X-linked dominant 8. d. Ocular involvement...
9. b. 5th 10. a. Autosomal recessive 11. a. Slit lamp... 12. c. Tuberous sclerosis
13. a. Iris nodule
172 Jaypee’s Triple A
14. Adenoma sebaceum is a feature of: (AIIMS Nov 05) e. Reiter’s syndrome Ref: Neena Khanna 4/e p43, 62
a. Neurofibromatosis (UPSC 02)
b. Tuberous sclerosis
c. Xanthomatosis E. MISCELLANEOUS
d. Incontinentia pigmenti Ref: Neena Khanna 4/e p33
22. Treatment of choice in solar keratosis: (DP PGMEE 2010)
15. Babloo a 4 year male presents with history of seizures. On
a. Methotrexate
examination there is hypopigmented patches on face &
b. Topical 5 FU
mental retardation. Most probable diagnosis is:
c. Topical mechlorethamine
a. Neurofibromatosis (AIIMS 2000)
d. Topical steroids Ref: Roxburgh 18/e p221
b. Tuberous sclerosis
c. Sturge Weber syndrome 23. Erythema marginatum is seen in: (Bihar 05, Kerala 97)
d. Incontinenta pigmenti Ref: Neena Khanna 4/e p32, 33 a. Drug reactions
b. Typhoid fever
16. Ash leaf maculae is found in:
c. Enteric fever
a. Tuberous sclerosis (Jharkhand 05)
d. Rheumatic fever Ref: Harrison 18/e p2754
b. Neurofibromatosis
c. Lymphangioma 24. Rose spot are seen in: (Kerala 98, UP 06)
d. None Ref: Neena Khanna 4/e p33 a. Typhus fever
b. Typhoid fever
17. Koenen’s periungual fibromas are seen in more than 50% of
c. Enteric fever
cases with: (JIPMER 02)
d. Rheumatic fever Ref: Harrison 18/e p1276
a. Tuberous sclerosis
b. Sturge weber syndrome 25. Common sites of mongolian spot are: (PGI Dec 08)
c. Alaxia telangiectasia a. Face
d. Neurofibromatosis Ref: Neena Khanna 4/e p33 b. Neck
c. Lumbo sacral area
18. All are true regarding tuberous sclerosis except:
d. Leg
a. Autosomal dominant sporadic transmission
e. Thigh Ref: Roxburgh 18/e p204
b. Vogt triad of epiloia (PGI June 08)
c. Café au lait macules exclude the diagnosis 26. Which of the following condition resolves spontaneously
d. Fibrous facial plaque in an infant: (PGI Dec 08)
e. Stippled confetti spots. a. Erythema toxicum
Ref: Neena Khanna 4/e p32-34; Harrison 18/e p891, 410, 2161 b. Mongolian spot
c. Lymphoma
d. Milia
D. ICHTHYOSIS e. Port wine stain
27. Fine reticular pigmentation with palmar pits are seen in:
19. A male child with cryptorchidism presents with large (AIIMS May 2011)
black scales on body flexures. Skin biopsy showed hyper a. Dowling-degos disease
granulosis & steroid sulfatase deficiency. Probable b. Rothmund thomson syndrome
diagnosis is: (SGPGI 08) c. Cockayane syndrome
a. Icthyosis vulgaris d. Bloom’s syndrome
b. Icthyosis lamellar Ref: Rook’s 8/e p15.94-15.95, 58.21/22, 52.41, 15.76 – 15.82;
c. X linked icthyosis nigra Fitzpatrick 7/e p1324-19; IJD vol 62 p25-29
DERMATOLOGY

d. Nonbullous icthyosiform erythroderma


28. Frei’s test is diagnostic of:
Ref: Neena Khanna 4/e p21
a. Lymphogranuloma venerum
20. Icthyosis is associated with: (UP 06, CUPGEE 96) b. Lymphogranuloma inguinale
a. Hodgkins disease c. Donovanosis
b. AIDS d. Soft chancre
c. Hypothyroidism Ref: Neena Khanna 4/e p310, 311
d. All
29. “Earliest” feature of tuberous sclerosis is:
Ref: Neena Khanna 4/e p21
a. Angiofibroma
21. Keratomdrema is/are seen in: (PGI May 11) b. Shagreen patch
a. Pemphigus c. Ash leaf spot
b. Pityriasis rosea d. Neurofibroma
c. Pityriasis rubra pilaris Ref: Neena Khanna 4/e p33
d. Dermatitis herpetiformis

Ans. 14. b. Tuberous sclerosis 15. b. Tuberous sclerosis 16. a. Tuberous sclerosis 17. a. Tuberous sclerosis
18. c. Cafe au lait... 19. c. X linked... 20. d. All 21. c. Pityriasis rubra...
22. b. Topical 5 FU 23. d. Rheumatic fever 24. c. Enteric fever 25. c, d and e
26. a, b and d 27. a. Dowling-degos 28. a. Lymphogranuloma... 29. c. Ash leaf spot
12. MALIGNANT AND DEFICIENCY
DISEASES OF SKIN

A. Malignant Diseases of Skin


B. Deficiency Diseases of Skin
174 Jaypee’s Triple A

MALIGNANT AND DEFICIENCY DISEASES OF SKIN (QUESTIONS)

A. MALIGNANT DISEASES OF SKIN 9. Acanthosis nigricans in old patient usually indicate:


a. Skin Disorder (Jharkhand 04, DNB 06)
1. Which naevi commonly predispose to malignant melanoma? b. Malignancy
(Delhi 2009 Feb) c. Senile Brain
a. Dermal d. Usually found in negro
b. Junctional Ref: Neena Khanna 4/e p361
c. Congenital 10. False regarding acanthosis nigricans is: (PGI 07)
d. Lentigo Ref: Harrison 18/e p724 a. Prognostic indicator
2. Least malignant melanoma is: (J & K 2010) b. Velvety plaques on dirty hyperpigmentation
a. Nodular c. Confirmatory sign of malignancy
b. Lentigo maligna d. Gastric adenocarcinma is commonest malignant
c. Superficial spreading association
d. Amelanotic e. Indicate hyperandrogenic state
Ref: Harrison 18/e p724 Ref: Neena Khanna 4/e p361
3. Kaposi’s sarcoma is caused by: (Raj 2009) 11. True about acanthosis nigricans: (PGI 03)
a. HHV-5 a. Most commonly seen in obesity
b. HHHV-5 b. Seen in axilla
c. HHV-7 c. It signifies internal malignancy
d. HHV-8 Ref: Neena Khanna 4/e p380 d. It is associated with insulin resistance
4. Skin markers of internal malignancy are/is: (PGI Nov 11) e. Seen in old age
a. Acanthosis nigricans Ref: Neena Khanna 4/e p30-37; Roxburg 18/e p294
b. Migratory necrolytic erythema 12. Mycosis fungoides which is not true: (AIIMS Nov 2006)
c. Necrobiosis lipodica a. It is the most common form of cutaneous lymphoma
d. Bullous pemphigoid b. Pautrier’s microabscess
e. Dermatomyositis c. Indolent course and good prognosis, easily amenable to
Ref: Neena Khanna 4/e p392-395 treatment
5. Multiple sebaceous tumors are seen in: (AI 2011) d. Erythroderma seen and spreads to peripheral circulation
a. Gardner’s syndrome Ref: Neena Khanna 4/e p377-379
b. Cowden’s syndrome 13. Pautrier’s micro-abscess is a histological feature of:
c. Carney complex a. Sarcoidosis (AIIMS Nov 05)
d. Muir-torre syndrome b. TB
Ref: Harrison 18/e p417 c. Mycosis fungoides
6. Predisposing factors for skin cancer are: (PGI 02) d. Ptyriasis lichenoides chronica
a. Lichen planus Ref: Neena Khanna 4/e p378
b. Bowen’s disease 14. Which of the following statements about mycosis fungoides
c. Psoriasis is not true? (AI 2007)
d. Behcet’s disease a. It is the most common skin lymphoma
DERMATOLOGY

e. UV rays Ref: Neena Khanna 4/e p359; Internet b. Pautrier’s microabscesses are common
7. Actinic keratosis is seen in: (AIIMS May 02) c. It has a indolent course and good prognosis.
a. Basal cell carcinoma d. It presents with diffuse erythroderma.
b. Squamous cell carcinoma Ref: Neena Khanna 4/e p377-379
c. Malignant melanoma 15. Total skin electron irradiation is used for treatment of:
d. Epithelial cell carcinoma Ref: Neena Khanna 4/e p359 (AI 12)
8. Which is related to sunlight exposure: (AIIMS May 12) a. Sezary syndrome
a. Actinic keratosis b. Mycosis fungoides
b. Molluscum contagiosum c. Psoriasis
c. Icthyosis d. Brain metastasis of skin cancer
d. Basal cell carcinoma Ref: Neena Khanna 4/e p379; Roxburg 18/e p237
Ref: Neena Khanna 4/e p361; Harrison 18/e p442, 731

Ans. 1. b. Junctional 2. b. Lentigo Maligna 3. d. HHV-8 4. a, b, d and e


5. d. Muir-torre syndrome 6. a, b and e 7. b. Squamous cell... 8. a. Actinic keratosis
9. b. Malignancy 10. c. Confirmatory... 11. a, b, c and d 12. c. Indolent course and...
13. c. Mycosis fungoides 14. c. It has a indolent... 15. b. Mycosis fungoides
Malignant and Deficiency Diseases of Skin 175
16. All are true about langerhans’ histocytosis except? 20. Mycosis fungoides is a:
a. Common before 3 years of age (AIIMS May 09) a. Cutaneous T-cell lymphoma
b. Letterer siwe disease is systemic manifestation b. T-cell and B-cell lymphoma
c. Radiosensitive c. B cell Lymphoma
d. Testis is commonly involved d. Derived from stem cell Ref: Neena Khanna 4/e p377
Ref: Neena Khanna 4/e p379 21. Birbeck granules are seen in
17. All are true regarding LCH except: (PGI 07) a. Tuberous sclerosis
a. Papular trunk lesions with scaling & crusting b. Langerhans cell histiocytosis
b. Ulcerative nodules on mucosa c. Paget’s disease
c. Calvarial defect with map lesions & floating teeth d. Neurofibromatosis Ref: Internet
d. CD – 68 positive
e. Polyostotis bone lesions t/t by polychemotherapy.
Ref: Neena Khanna 4/e p379 B. DEFICIENCY DISEASES OF SKIN
18. Rx of refractory histiocytosis? (AIIMS Nov 08)
22. All are true about Acrodermatitis enteropathica except:
a. Cladarabine
(AIIMS Nov 08, May 011)
b. High dose MTX
a. Zn level (low serum zinc level)
c. High dose cytosine arabinoside
b. Reverse with Zn supplement
d. Fludarabine
c. Triad of acral dermatitis, dementia & diarrhea
Ref: National Cancer Institute www.cancer.gov;
d. AR
Fitzpatrick 7/e p1414-24; CMDT 2009 p965;
Ref: Neena Khanna 4/e p393
Neena Khanna 3/e p323; Journal of Pediatric Onchology; Robbins 7/e
p701-02, 199-200 23. Dermatitis and alopecia are due to deficiency of:
a. Zinc (SGPGI 2001)
19. Child presents with linear verrucous plaques on the trunk
b. Molybodenum
with vacuolization of keratinocytes in s. spinosum and s.
c. Magnesium
granulosum. Diagnosis is: (AIIMS Nov 08, May 11)
d. Calcium
a. In continenta pigmenti
Ref: Neena Khanna 4/e p393
b. Delayed hypersensitivity reaction
c. Linear/verrucous epidermal nevus 24. Zinc is specifically used for:
d. Linear darier’s disease a. Acrodermatitis enteropathica
Ref: Fitzpatrick’s 7/e p1056-58, 631, 32, 443, 435; b. Urticaria pigmentosa
Thomas Habif 4/e p713-14; Department of DNYU, online journal 9(4)15 c. Tuberous sclerosis
d. Pemphigus
Ref: Neena Khanna 4/e p393

DERMATOLOGY

Ans. 16. d. Testis is commonly... 17. d and e 18. a. Cladarabine 19. c. Linear/verrucous
20. a. Cutaneous T-cell... 21. b. Langerhans cell... 22. c. Triad of acral... 23. a. Zinc
24. a. Acrodermatitis...
Section B

PRACTICE QUESTIONS
(Comprising of Questions from Recent Exams and
NEET Pattern Questions)
DERMATOLOGY
Practice Questions

1. Which of the following is/are not the cutaneous 8. A 60-years-old man presented with itchy tense blisters on
manifestation of diabetes mellitus: (PGI May 2013) normal looking skin and urticarial rash. Investigation done
a. Necrobiosis lipoidica for the diagnosis: (AIIMS May 2013)
b. Diabetic bullae a. Direct immunofluorescence
c. Shin spots b. Indirect immunofluorescence
d. Calcinosis cutis c. Histopathology
e. Angiokeratoma d. Cytopathology Ref: Harison 18/e p426-428
Ref: Neena Khanna 4/e p381-383 9. A 10-years-old boy presented with painful boggy swelling
2. Microabscess is /are seen in: (PGI May 2013) of scalp, multiple sinuses with purulent discharge, easily
a. Psoriasis pluckable hairs and lymph nodes enlarged in occipital
b. Lichen planus region. Which one of the following would be most helpful
c. Pityriasis versicolor for diagnostic evaluation? (AIIMS May 2013)
d. Pityriasis rosea a. Bacterial culture
e. Mycosis fungoides b. Biopsy
Ref: Neena Khanna 4/e p40-377 c. KOH mount
3. Drug used for pediculosis is/are: (PGI May 2013) d. Patch test Ref: Neena Khanna 4/e p283-288
a. Malathion 10. The Ridley-Jopling classification for leprosy is based on
b. Permethrin which of the following parameters? (AIIMS May 2013)
c. Ivermectin a. Clinical, bacteriological, immunological
d. Diethylcarbazine b. Histopathological, clinical, therapeutic
e. Nitrate c. Histopathological, epidemiological, therapeutic
Ref: Neena Khanna 4/e p338-339 d. Histopathological, clinical, epidemiological
4. Causative factor of acne include(s): (PGI May 2013) Ref: Rox burg18/e p49, 50; Fitz Patrick 7/e p1786-1796
a. Hypersecretion of sebum 11. Dry ice is? (DNB 2013)
b. IgE level a. Methane hydrate
c. Follicular duct hypercornification b. Liquid nitrogen
d. Colonisation of propionibacterium acnes c. Solid carbon
e. IGF-I d. Frozen water Ref: Rook’s 8/e p77, 39
Ref: Neena Khanna 4/e p110-122 12. Linear deposition of IgG and C3 in lamina lucida is seen
5. A young 8-years-old boy with multiple, small, pin point, in? (DNB 2013)
shiny, popular lesions on dorsal aspect of hand, forearms a. Dermatitis herpetiformis
and his penis also. Diagnosis: (AIIMS May 2013) b. Pemphigus vulgaris
a. Molluscum cantagiosum c. Pemphigus folicaeous
b. Scabies d. Bullous pemphigoid Ref: Neena Khanna 4/e p77, 78
c. Lichen planus 13. The superficial veins are found in which layer of skin?
d. Lichen nitidus Ref: Neena Khanna 4/e p60 a. Epidermis (NEET Pattern Question)
6. An old patient presents with painful red vesicular eruption b. Dermis
confined to T3 dermatome. Diagonsis: (AIIMS May 2013) c. Subdermal
a. Varicella zoster d. Muscles Ref: Internet
b. Herpes simplex 14. Antibody against BP120 and BP180 seen in?
c. HIV a. Pemphigus vulgaris (DNB 2013)
d. Dermatitis Ref: Neena Khanna 4/e p275 b. Bullous pemphigoid
7. All of the following drugs can lead to SLE like reaction c. Dermatitis herpetiformis
except? (AIIMS May 2013) d. Linear IgA disease Ref: Neena Khanna 4/e p77
a. Hydralazine 15. Wickham’s striae is seen in? (DNB 2013)
b. Penicillin a. Lichen planus
c. Isoniazid b. Psoriasis
d. Sulphonamide c. Prurigo
Ref: Neena Khanna 4/e p223, 224 d. DLE Ref: Neena Khanna 4/e p57

Ans. 1. d and e 2. a and e 3. a and b 4. a, c, d and e


5. d. Lichen nitidus 6. a. Vericella zoster 7. b. Penicillin 8. a. Direct...
9. c. KOH mount 10. a. Clinical, bacteriological... 11. c. Solid Carbon 12. d. Bullous pemphigoid
13. a. Epidermis 14. b. Bullous Pemphigoid 15. a. Lichen planus
178 Jaypee’s Triple A
16. Sawtooth rete ridges are seen in? (DNB 2013) c. Stratum corneum
a. Erythrasma d. Stratum granulosum Ref: Internet
b. Psoriasis 27. Heavy bacterial colonisation of tongue presents as?
c. Darier’s disease a. Smooth tongue (DNB 2013)
d. Lichen planus Ref: Rook’s 8/e p41.14; Internet b. Fissured tongue
17. Vagabond’s disease is caused by? (DNB 2013) c. Black tongue
a. Scabies d. Median rhomboid glossitis Ref: Internet
PRACTICE Qs – Dermatology

b. Herpes genitalis 28. Coarse pitting of nails is seen in? (DNB 2013)
c. Pediculosis corporis a. Psoriatic arthirits
d. Ant bite reaction Ref: Neena Khanna 4/e p339 b. Dermatitis herpetiformis
18. Boil is an infection of: (NEET Pattern Question) c. Bullous pemphigoid
a. Sweat gland d. Pemphigus vulgaris Ref: Neena Khanna 4/e p45
b. Hair follicle 29. Eccrine glands are involved in: (DNB 2013)
c. Subcutaneous tissue a. Bromhidrosis
d. Scalp Ref: Internet b. Miliaria
19. Obliteration of apocrine duct leads to? (DNB 2013) c. Hidradenitis suppurativa
a. Fordyce’s disease d. Fox– Fordyce disease Ref: Neena Khanna 4/e p126
b. Fox fordyce’s disease 30. Which of the following is not a nevus of melanocyte:
c. Moll’s gland a. Mongolian spot (DNB 2013)
d. Pearly benign papules Ref: Rook’s 7/e p45.23 b. Nevus of Ito
20. Wavelength of carbon dioxide laser is? (DNB 2013) c. Nevus of ota
a. 1064 nm d. Becker nevus Ref: Neena Khanna 4/e p357
b. 1082 nm 31. Wood lamp wavelength: (DNB 2013)
c. 2940 nm a. 360-385
d. 10600 nm Ref: Rook’s 8/e p78.6 b. 280-320
21. Koenen’s tumor is seen in? (DNB 2013) c. 400-450
a. Tuberous sclerosis d. 450-500 Ref: Neena Khanna 4/e p14
b. Neurofibromatosis 32. Hidradenitis suppurativa is a disease of? (DNB 2013)
c. Sturge weber syndrome a. Apocrine glands
d. Ichthyosis Ref: Neena Khanna 4/e p32 b. Eccrine glands
22. Coral red colour on wood’s lamp is seen in? c. Holocrine glands
a. P. versicolor (DNB 2013) d. None of the above Ref: Neena Khanna 4/e p127, 128
b. Amyloidosis 33. Cafe au lait spots are seen in? (DNB 2013)
c. Melasma a. NF
d. Erythrasma Ref: Neena Khanna 4/e p242, 243 b. Gardner syndrome
23. DRESS syndrome is associated with all except: c. Cockayne syndrome
a. Drug reaction (DNB 2013) d. Down syndrome Ref: Neena Khanna 4/e p34
b. Eosinophilia 34. Soft sore is caused by? (DNB 2013)
c. Myocarditis a. H. ducreyi
d. Encephalitis b. Calymmatobacterium granulomatis
24. Norwegian scabies is seen in? (DNB 2013) c. Chlamydia trachomitis
a. Children d. T. Pallidum Ref: C.P Baweya 4/e p319
b. Pregnant women 35. Drug not causing exanthematous skin eruption:
c. Patients on chemotherapy a. Phenytoin (DNB 2013)
d. Infants Ref: Neena Khanna 4/e p343 b. Hydrocortisone
25. Groove sign is seen in? (DNB 2013) c. Ampicillin
a. Chancroid d. Phenylbutazone Ref: KD’Tripathi
b. Syphilis 36. Not a primary skin disease: (DNB 2013)
c. LGV a. Lichen planus
d. Psoriasis Ref: Neena Khanna 4/e p310 b. Psoriasis
26. Melanocytes are located in which layer? (DNB 2013) c. Reiter’s disease
a. Stratum malphigii d. Vitiligo Ref: Neena Khanna 4/e p43
b. Stratum basale

Ans. 16. d. Lichen planus 17. c. Pediculosis... 18. b. Hair follicle 19. b. Fox fordyce’s...
20. d. 10600 nm 21. a. Tuberous sclerosis 22. d. Erythrasma 23. d. Encephalitis
24. c. Patients on... 25. c. LGV 26. b. Stratum basale 27. c. Black tongue
28. a. Psoratic arthirits 29. b. Miliaria 30. d. Becker nevus 31. b. 280-320
32. a. Apocrine glands 33. a. NF 34. a. H ducreyi 35. b. Hydrocortisone
36. c. Reiter’s disease
Practice Questions 179
37. Hereditary angioneurotic edema false is: 47. Phrynoderma is NOT due to deficiency of:
a. C4 for screening (DNB 2013) a. EFA (DNB 2013)
b. Deficiency of C1 inhibitor b. Vit A
c. Excessive production of C2 kinin c. Vit C
d. C3 level are reduced Ref: Neena Khanna 4/e p181 d. Vit D Ref: Neena Khanna 4/e p391
38. Desmosomes are help is connecting: (DNB 2013) 48. At what wavelength UVB are rays given in phototherapy?
a. Keratinocytes a. 311 (NEET Pattern Question)

PRACTICE Qs – Dermatology
b. Melanocytes b. 321
c. Dermis and epidermis c. 512
d. Langerhans cells Ref: Internet d. 127 Ref: Neena Khanna 4/e p52
39. True about atopic dermatitis are all except: 49. Which of the following is untrue regarding piebaldism:
a. Pruritus (DNB 2013) a. Autosomal dominant condition (DNB 2013)
b. Scratching b. Amelanotic skin associated with a white forelock
c. Mica like scales c. Islands of normal of hypermelanotic skin
d. Eczema Ref: Neena Khanna 4/e p91 d. Usually improves with age
40. Most common side effect of oral isotretinoin given for acne Ref: Neena Khanna 4/e p149
vulgaris is: (NEET Pattern Question) 50. All the following are true about Pemphigus except:
a. Dry skin a. Positive nikolsky’s sign (DNB 2013)
b. Diarrhea b. Intraepidermal bullae
c. Teratogenicity c. Mucosa not involved
d. Drug reaction Ref: K.D’Tripathi 7/e p895 d. Acantholysis is present Ref: Neena Khanna 4/e p71, 73
41. The nail change pterygium is seen in: 51. Acrodermatitis entropathica is seen with deficiency of:
a. Psoriasis (NEET Pattern Question) a. Zinc (DNB 2013)
b. Atopic dermatitis b. Iron
c. Lichen planus c. Copper
d. Lichen simplex chronicus Ref: Neena Khanna 4/e p143 d. Vit A Ref: Neena Khanna 4/e p393
42. Exanthema subitum is caused by: (DNB 2013) 52. Sebum is not composed of? (NEET Pattern Question)
a. HSV a. Cholesterol
b. HPV b. Wax
c. HIV c. Glycerides
d. HCV Ref: Internet d. Propylene Ref: Rox burg 18/e p10
43. Characteristic lesion in scabies: (DNB 2013) 53. Not a feature of scabies: (DNB 2013)
a. Burrows a. Burrows are seen in stratum corneum
b. Vesicle b. Itching is more severe at night
c. Papule c. Family history is found
d. Pastule Ref: Neena Khanna 4/e p341 d. Fever is a common finding
44. Stage 1 cutaneous T cell lymphoma treatment is: Ref: Neena Khanna 4/e p341
a. PUVA (DNB 2013) 54. Oculoorogenital ulcers are a feature of: (DNB 2013)
b. Biological response modifiers a. Behcet disease
c. Systemic chemotherapy b. Lichen planus
d. Extracorporal photophersis c. SLE
Ref: Internet; Neena Khanna 4/e p418 d. Psoriasis Ref: Neena Khanna 4/e p238, 239
45. In pemphigus vulgaris blisters are located in? 55. Lucio reaction is seen in: (DNB 2013)
a. Sub epidermal (DNB 2013) a. TB
b. Intra epidermal b. Leprosy
c. Subdermal c. Syphillis
d. Subfascial Ref: Neena Khanna 4/e p71 d. LGV Ref: Internet
46. Dermatophytosis is not: (DNB 2013) 56. ENL lepra reaction is seen in: (NEET Pattern Question)
a. Scaly a. TT
b. Itchy b. BT
c. Superficial c. BB
d. Subdermal Ref: Neena Khanna 4/e p283 d. Lepromatous leprosy Ref: Neena Khanna 4/e p256, 257

Ans. 37. d. C3 level are... 38. a. Keratinocytes 39. c. Mica like scales 40. c. Teratogenicity
41. c. Lichen planus 42. a. HSV 43. a. Burrows 44. a. PUVA
45. b. Intra epidermal 46. a and b 47. c and d 48. a. 311
49. d. Usually... 50. c. Mucosa not involved 51. a. Zinc 52. c and d
53. a and b 54. a. Behcet disease 55. b. Leprosy 56. d. Lepromatous leprosy
180 Jaypee’s Triple A
57. Dermatomyositis test of choice: (DNB 2013) 67. In sebaceous glands accumulation of sebum leads to:
a. CPK a. Milia (DNB 2013)
b. 24hr creatine in urine b. Acne
c. SGOT c. Epidermoid cyst
d. SGPT Ref: Neena Khanna 4/e p227 d. Miliaria Ref: Neena Khanna 4/e p109, 110
58. STD with pseudobubo formation: (DNB 2013) 68. Anti leprosy drug used in pauci bacillary are:
a. Granuloma inguinale a. R + D (NEET Pattern Question)
PRACTICE Qs – Dermatology

b. LGV b. R + D + C
c. Chancroid c. R + C
d. Syphilis erode through skin to produce ulcer. d. D + C Ref: Neena Khanna 4/e p267
Ref: Neena Khanna 4/e p309 69. Not true about skin tag: (DNB 2013)
59. Immune reactants deposited at dermo-epidermal junction a. Associated with seborrhoeic keratosis
in positive lupus test: (DNB 2013) b. Pedunculated
a. IgG only c. Most common site is neck and axilla
b. IgG with compliments d. Premalignant Ref: Neena Khanna 4/e p352
c. IgM only 70. Which one is the drug of choice for moderate Lepra-2
d. IgG with IgM Ref: Neena Khanna 4/e p225 reaction in 40 years female? (NEET Pattern Question)
60. What is not true about erythema multiforme: a. Thalidomide
a. Associated with H simplex (DNB 2013) b. Methotrexate
b. Involves distal parts of extremities c. Cyclosporine
c. No vesicle d. Steroids Ref: Neena Khanna 4/e p268
d. Target lesions Ref: Neena Khanna 4/e p174 71. Tzank smear is positive in: (DNB 2013)
61. Spaghetti and meat ball appearance is seen in: a. Herpes simplex
a. Dermatophytes (DNB 2013) b. Psoriasis
b. Asperguillus c. LP
c. Pityriasis versicolor d. Warts Ref: Neena Khanna 4/e p15
d. Candida Ref: Neena Khanna 4/e p290, 291 72. Loss of Intercellular cohesion between keratinocytes is
62. Best diagnostic test for fungal skin infection: called as: (DNB 2013)
a. KOH test (DNB 2013) a. Acanthosis
b. Diascopy b. Acantholysis
c. Wood’s lamp c. Keratinolysis
d. Patch test Ref: Neena Khanna 4/e p15 d. Spongiosis Ref: Internet
63. Rx for stage 1 mycosis fungoides: (DNB 2013) 73. Tomb stone appearance is seen in? (DNB 2013)
a. PUVA a. Bullous pemphigoid
b. Biological response modifiers b. Pamphigus vulgaris
c. Systemic chemotherapy c. Linear IgA disease
d. Extracorporeal photopheresis d. Dermatitis herpetiformis
Ref: Neena Khanna 4/e p377 Ref: Neena Khanna 4/e p72; Roxburg 18/e p103
64. Non scarring loss of hair: (DNB 2013) 74. Target Lesion of acral areas seen in? (DNB 2013)
a. Alopecia areata a. Stevens Johnson syndrome
b. Lichen planus b. Erythema Multiforme
c. SLE c. TEN
d. Dissecting folliculitis Ref: Neena Khanna 4/e p129 d. Lichen planus Ref: Neena Khanna 4/e p339
65. True about lepromatous leprosy: (DNB 2013) 75. Trachyonychia: (NEET Pattern Question)
a. Only 3 cutaneous lesions a. 10-10 nail dystrophy
b. Lepromin test highly positive b. 5-5 nail dystrophy
c. Thickened nerve roots c. 15-15 nail dystrophy
d. ENL in > 50% cases Ref: Neena Khanna 4/e p259 d. 20-20 nail dystrophy Ref: Neena Khanna 4/e p413
66. Pterygium of nail is seen in: 76. 2-year-old with rash on extensor surface, diagnosis:
a. Lichen planus (DNB 2013) a. Infantile eczema (DNB 2013)
b. Psoriasis b. Atopic dermatitis
c. Tinea unguium c. Contact dermatitis
d. Alopecia areata Ref: Neena Khanna 4/e p60 d. Viral exenthema Ref: Neena Khanna 4/e p92

Ans. 57. a. CPK 58. a. Granuloma inguinale 59. a. IgG only 60. a. Associated with H simplex
61. c. Pityriasis versicolor 62. a. KOH test 63. a. PUVA 64. a. Alopecia areata
65. a and c 66. a. Lichen planus 67. b. Acne 68. a. R + D
69. d. Premalignant 70. a. Thalidomide 71. a. Herpes simplex 72. c. Keratinolysis
73. b. Pamphigus Vulgaris 74. b. Erythema Multiforme 75. d. 20-20 nail... 76. b. Atopic dermatitis
Practice Questions 181
77. S.lucidiium layer is present between which layers of skin? 87. Leprosy does not affect: (FMGE March 2013)
a. S. corneum and S. granulosum (NEET Pattern Question) a. Testis
b. S. granulosum and S. spinosum b. Uterus
c. S. spinosum and S.basalis c. Eye
d. S. basalsis and dermis Ref: Roxburg 18/e p1 d. Nerve Ref: Roxburg 18/e p49, 50
78. Tzank smear is for: (DNB 2013) 88. In congenital dystrophic variety of epidermolysis bullosa,
a. HZV mutation is seen in the gene coding for:

PRACTICE Qs – Dermatology
b. Psittacosis a. Laminin 4 (FMGE March 2013)
c. Cryptococcus b. Collagen type 7
d. Ricketssia Ref: Neena Khanna 4/e p15, 16 c. Alpha 6 integerin
79. In pemphigus vulgaris, tzank smear shows: d. Keratin 14 Ref: Neena Khanna 4/e p31
a. Acantholytic cells (DNB 2013) 89. Which mineral causes skin allergy: (FMGE March 2013)
b. Macrophages a. Magnesium
c. Fibroblasts b. Zinc
d. Neutrophills Ref: Neena Khanna 4/e p16 c. Copper
80. Pteryigum of nail is seen in: (DNB 2013) d. Nickel Ref: Neena Khanna 4/e p101
a. Lichen planus 90. Protective from ultra-violet rays is: (FMGE March 2013)
b. Psoriasis a. Keratinocytes
c. Tinea unguium b. Melanocytes
d. Alopecia areata Ref: Neena Khanna 4/e p60 c. Langerhans cells
81. 25-year-old male has fever and malaise since 2 weeks, d. Merker cells Ref: Roxburg 18/e p6
arthritis of ankle joint and tender erthymatous nodules over 91. Gottron’s papules/spots are pathognomic of:
the skin. Diagnosis is: (DNB 2013) (FMGE March 2013)
a. Erythema nodosum a. Myasthenia gravis
b. Hensen’s disease b. Dermatomyositis
c. Weber-christian disease c. Dermatitis herpetiformis
d. Urticarial vasculitis Ref: Neena Khanna 4/e p188, 189 d. Atopic dermatitis Ref: Neena Khanna 4/e p228
82. Ichy purple papule followed by hyperpigmentation on 92. A 25-year-old man present with recurrent episodes of
resolution, is seen in: (DNB 2013) flexural eczema, contact urticaria, recurrent skin infection
a. Addison’s disease and severe abdominal cramps and diarrhea upon taking sea
b. DM foods. He is suffering from: (FMGE March 2013)
c. Hypothyrodism a. Seborrheic dermatitis
d. Lichen planus Ref: Neena Khanna 4/e p56, 57 b. Atopic dermatitis
83. Tzank smear: (DNB 2013) c. Airborne contact dermatitis
a. HSV d. Nummular dermatitis
b. Psittacosis Ref: Neena Khanna 4/e p91-96
c. Cryptococcus 93. A child born with nasal discharge and rash all over the
d. Ricketssia Ref: Internet; Neena Khanna 4/e p15, 16 body. The diagnosis is: (FMGE March 2013)
84. Pomphylox affects: (DNB 2013) a. Roseola infantum
a. Palms & soles b. Congenital syphilis
b. Groin c. Erythema infectiosum
c. Scalp d. Rubella Ref: Neena Khanna 4/e p304
d. Trunk Ref: Neena Khanna 4/e p104 94. Which is not a sexually transmitted disease:
85. Characteristic lesion in scabies: (DNB 2013) (FMGE March 2013)
a. Burrows a. Chancroid
b. Vesicle b. Yaws
c. Papule c. Granuloma Inguinale
d. Pustule Ref: Neena Khanna 4/e p341 d. LGV Ref: Davidson 21/e p328
86. Phrynoderma is NOT due to deficiency of: 95. Genital warts are seen in: (FMGE March 2013)
a. EFA (DNB 2013) a. Syphilis
b. Vit A b. Leprosy
c. Vit C c. TB
d. Vit D Ref: Neena Khanna 4/e p391 d. Buschke lowenstein tumor Ref: Davidson 21/e p423

Ans. 77. a. S. corneum and S. ... 78. a. HZV 79. a. Acanthoytic cells 80. a. Lichen planus
81. a. Erythema nodosum 82. d. Lichen planus 83. a. HSV 84. a. Palms & soles
85. a. Burrows 86. c and d 87. b. Uterus 88. b. Collagen type 7
89. d. Nickel 90. b. Melanocytes 91. b. Dermatomyositis 92. b. Atopic dermatitis
93. b. Congenital syphilis 94. b. Yaws 95. d. Buschke Lowenstein...
182 Jaypee’s Triple A
96. Herald patch is caused by: (FMGE March 2013) 105. Staphylococcus aureus infection causes all of the following
a. Psoriasis disease except: (FMGE March 2013)
b. Leprosy a. Impetigo
c. Lichen planus b. Erysipelas
d. None of the above Ref: Neena Khanna 4/e p54-58 c. Ecthyma
97. Patch test read after: (FMGE March 2013) d. Scaldy skin syndrome Ref: Neena Khanna 4/e p245-248
a. 2 hrs 106. Which of the following is not photosensitive:
PRACTICE Qs – Dermatology

b. 2 weeks a. Porphyia (FMGE March 2013)


c. 2 months b. DLE
d. 2 days Ref: Neena Khanna 4/e p16 c. SLE
98. Munro micro abscess is seen in: (FMGE March 2013) d. Lichen planus Ref: Neena Khanna 4/e p56-58
a. Dermal tissue 107. Melasma seen during pregnancy is on: (FMGE March 2013)
b. Stratum basale a. Face
c. Stratum corneum b. Trunk
d. Stratum malpiphi Ref: Neena Khanna 4/e p48 c. Lower limb
99. Silver plaques are seen in: (FMGE March 2013) d. Back Ref: Roxburg 18/e p252
a. Psoriasis 108. A 23-years-old man present with recurrent episodes of
b. Vitiligo flexural eczema, contact urticarial, recurrent skin infections
c. Albinism with severe abdominal craps and diarrhea upon taking sea
d. Lichen sclerosis Ref: Neena Khanna 4/e p48 foods. He is suffering from: (FMGE March 2013)
100. All of the following are nail deformities including all a. Nummular dermatitis
except: b. Atopic dermatitis
a. Oncholysis c. Airborne disease
b. Pterygium d. Seborrheic dermatitis Ref: Neena Khanna 4/e p91
c. Beau’s lines 109. Hyperplasia of stratum corneum with normal maturity is
d. Longitudinal grooves Ref: Neena Khanna 4/e p58 seen in: (FMGE March 2013)
101. A 3-year-old child has eczematous dermatitis on extensor a. Lichenification
surfaces. His mother has a history of Bronchial asthma b. Parakeratosis
during her childhood. The likely diagnosis should be: c. Hyperkeratosis
a. Atopic dermatitis (FMGE March 2013) d. All of the above Ref: Neena Khanna 4/e p39
b. Contact dermatitis 110. Most important in diagnosis of Leprosy is:
c. Seborrhic dermatitis a. Evidence of neural involvement (FMGE March 2013)
d. Infantile eczematous dermatitis b. Hypopigmented patches
Ref: Neena Khanna 4/e p91, 92 c. Slit skin smears for AFB
102. A child born with nasal discharge and rash all over the d. Positive lepromin test Ref: Neena Khanna 4/e p267, 268
body. The diagnosis is: (FMGE March 2013) 111. Most common type of vitiligo in India is:
a. Roseola infantum a. Vitiligo universalis (FMGE March 2013)
b. Erythema infectiosum b. Acrofacial vitiligo
c. Rubella c. Vitiligo vulgaris
d. Congenital syphilis Ref: Neena Khanna 4/e p305 d. Segmental vitiligo Ref: Neena Khanna 4/e p151
103. Acantholysis is seen in: (FMGE March 2013) 112. A 34-year-old female presents with itchy, maculopapular
a. Pemphigus lesion in the inner aspect of wrists, ankles and forearms.
b. Psoriasis Her oral mucosa has a white lace like eruptions. What is the
c. Pemphigoid most probable diagnosis: (FMGE March 2013)
d. Steven johnson syndrome Ref: Neena Khanna 4/e p71 a. Lichen planus
104. Commonest variety of pemphigus: (FMGE March 2013) b. Eczema
a. Pemphigus vulgaris c. Pitryasis rosea
b. Pemphigus vegetans d. Psoriasis Ref: Neena Khanna 4/e p57
c. Pemphigus foliaceus
d. Pemphigus erythematosus Ref: Neena Khanna 4/e p71

Ans. 96. d. None of the above 97. d. 2 days 98. c. Stratum corneum 99. a. Psoriasis
100. c. Beau’s lines 101. a. Atopic dermatitis 102. d. congenital... 103. a. Pemphigus
104. a. Pemphigus vulgaris 105. b. Erysipelas 106. d. Lichen planus 107. a. Face
108. b. Atopic dematitis 109. c. Hyperkeratosis 110. c. Silt skin smeas... 111. c. Vitiligo Vulgaris
112. a. Lichen planus

S-ar putea să vă placă și